Вы находитесь на странице: 1из 144

DNS Lecture Objectives

Table of Contents
Neurological Diagnosis................................................................................................................... 2
Introduction to Neuropathology ................................................................................................. 5
Localizing Lesions in the Motor System................................................................................... 11
Localizing Somatosensory Lesions ........................................................................................... 17
Spinal Disorders............................................................................................................................. 20
Neuromuscular Disease ............................................................................................................... 25
Neuropathology of Peripheral Nerve ....................................................................................... 31
Localizing Lesions of the Visual Pathways.............................................................................. 38
Cerebral Blood Flow & Metabolism .......................................................................................... 42
Ischemic Stroke and Intracerebral Hemorrhage .................................................................. 44
Intracranial Pressure & Head Injury ........................................................................................ 50
Subarachnoid Hemorrhage and Cerebral Vascular Malformations ................................ 53
Pathology of Circulatory Diseases of the CNS......................................................................... 56
Seizures and Epilepsy ................................................................................................................... 61
Sleep Disorders .............................................................................................................................. 64
Coma, Persistent Vegetative State, & Death by Neuro Criteria ......................................... 69
Pathology of CNS Tumors ............................................................................................................ 75
Headache.......................................................................................................................................... 85
Neurobiology of Rehabilitation ................................................................................................. 89
Aphasia ............................................................................................................................................. 92
Neglect .............................................................................................................................................. 94
Multiple Sclerosis .......................................................................................................................... 96
Pathology of Diseases of Myelin.............................................................................................. 101
Movement Disorders and Therapy ........................................................................................ 111
Dementia ....................................................................................................................................... 120
Pathology of Neurodegenerative Disease ............................................................................ 124
CNS Infections .............................................................................................................................. 132
Pathology of CNS Infections ..................................................................................................... 136

Return to Table of Contents

Neurological Diagnosis
Define the levels of the neuroaxis and list the characteristic signs and symptoms
associated with a lesion at each one
Level of Nervous System
Dysfunction
Peripheral
Muscle
Myopathy
NMJ
Nerve
Neuropathy
Plexus
Plexopathy
Root
Radiculopathy
Cell body (anterior horn cell or dorsal
Lower motor neuron disease or
root ganglion)
ganglionopathy
Spinal Cord
Myelopathy
Posterior fossa
Cerebral hemispheres
Encephalopathy
(Subarachnoid space/meninges)
PNS Localization
o All will have lower motor neuron type motor loss:
Flaccid weakness
Atrophy
Hypotonia
May have: hyporeflexia, fibrillations (not visible, occur in individual
muscle fibers), or fasciculations (visible, discharge of whole axons)
o Myopathy
Motor only (proximal, symmetric)
Ex/ shoulders, hips; climbing stairs, getting out of low chair,
reaching overhead
Atrophy and reflex loss occur if severe
Ex/ polymyositis, muscular dystrophy
o Neuropathy
Single Nerve (ex/ carpal tunnel syndrome)
Motor, sensory (all modalities), reflex loss in distribution of
specific nerve
Polyneuropathy (ex/ diabetic polyneuropathy)
Motor, maybe sensory/autonomic (burning/pain)
Legs before hands; more prominent distally
Symmetric stocking-glove distribution
Axonal (loss of one reflex), demyelinating (loss of all reflexes)
o Plexopathy
Mixed nerve and root deficit: LMN weakness, sensory & reflex loss
Brachial (C5-T1, usually upper or lower parts, Horners with C8
involvement) trauma
Lumbosacral (T12-S4, bowel/bladder with pudendal involvement)
neoplasm
Return to Table of Contents

o Radiculopathy (disc herniation C5-T1 & L4-S1 common)


Segmental (dermatome) pain, sensory, LMN motor, and reflex loss
Weakness/atrophy in distribution of affected spinal root (myotome)
o LMN Disease (anterior horn cell spinal muscular atrophy, polio)
Motor loss only (no sensory loss)
Diffuse or single limb; proximal & distal LMN weakness and reflex loss
Involvement of paraspinal muscles
o Sensory Ganglionopathy (DRG paraneoplastic, Sjogrens, idiopathic)
Sensory loss to all modalities (proprioception is most prominent)
Usually proximal & distal (may involve face/trunk)
Often asymmetric; diffuse reflex loss
Myelopathy (spinal cord spondylosis is common, MS, trauma)
o UMN dysfunction BELOW the level of lesion
Interruption of corticospinal tract no atrophy)
Spasticity, hyperreflexia, extensor plantar response
o LMN weakness AT level of lesion (if gray matter involved)
o Sensory level or dissociated sensory loss
Posterior Fossa
o Brainstem
CN deficits
Impaired consciousness, impaired respiration
Crossed/bilateral motor or sensory deficits (left facial, right body
sensory loss w/ left lateral medullary lesion)
Quadriplegia with pontine lesion
o Cerebellum
Signs during movement or sustained posture (incoordination, ataxia,
nystagmus, tremor)
No weakness, sensory loss, reflex change
Vermis = truncal deficit; Hemisphere = appendicular (arms/legs)
Supratentorial
o Thalamus
(DENSE) hemisensory loss and/or pain
Impaired consciousness, memory disturbance, hemiataxia,
aphasia/neglect [[symptoms may mimic cortical defect]]
Dysfunction most commonly due to stroke (vertebrobasilar)
o Basal ganglia (Parkinsons disease, Huntingtons chorea)
Slow movement (bradykinesia)
Signs at rest: muscle rigidity, involuntary movements
Chorea sudden, brief, irregular jerks (loss of caudate)
Athetosis slow, writhing movements (caudate, putamen, GP,
subthalamus)
Dystonia slow, contorting posturing (caudate, putamen,
thalamus)
Ballismus flinging movement of limb or half of body
(subthalamus, caudate, putamen, GP, thalamus)
Tremor
Return to Table of Contents

o Cerebral cortex
UMN loss: hemiparesis/spasticity/hyperreflexia
Hemisensory deficits (different pattern than thalamic, integrative
sensory disturbance or reduction in single modality)
Hemianopia, aphasia/neglect, dementia, seizures
Subarachnoid space/meninges (meningitis, subarachnoid hemorrhage)
o Headache, neck stiffness
o Cranial nerve deficits
o Altered level of consciousness

Distinguish among focal, multifocal, and diffuse localization and relate these to
pathophysiology
Focal = single lesion (ex/ tumor, stroke)
Multifocal = more than 1 discrete lesion (ex/ MS, mononeuritis multiplex)
Diffuse = generalized process (ex/ encephalopathy, polyneuropathy)
o System Specific = diffuse dysfunction of specific system or pathway (ex/
anterior horn cell disease)
Multiple Symptoms consider common anatomy, vascular supply, system, or
pathophysiology
Use the presence or absence of neighborhood signs to support a proposed
neuroanatomical localization
Ex/ facial nucleus in the pons and exits via cerebellopontine angle
o Expect ipsilatral LR palsy with pons damage
o Deafness with cerebellopontine angle deficit
When given a clinical scenario, determine the time-intensity profile of the
neurological symptoms and how it relates to the pathophysiology of the disease
process
Acute (min-hrs): metabolic dysfunction cerebral ischemia or seizure
Subacute (days-weeks): expanding lesion tumor or abscess
Recurrent-remittent (episodic with recovery) multiple sclerosis or migraine
Chronic progressive (months-years): slow-growing tumor or degenerative
Alzheimers or Parkinsons
Distinguish among positive, negative, secondary, and behavioral symptoms
Negative: reduction or loss of function (weakness, numbness, blindness)
Positive: exaggeration of a physiologic phenomenon (chorea, seizure, tingling, visual
hallucinations)
Secondary: mass effect (edema, herniation, obstruction of CSF)
Behavioral: changes in personality/behavior (dementia, psychosis, neglect)

Return to Table of Contents

Introduction to Neuropathology
Identify the basic cell types of the nervous system and their characteristic
histopathologic reactions
Neurons (>500 sub-types in the cerebral cortex alone)
o Nonpathologic Neuronal Inclusions: lipofucsin (orange), neuromelanin (dark
brown, found in substantia nigra and locus ceruleus residua of
catecholamine metabolism)
o Pathologic:
Alzheimers Neurofibrillary Tangles: silver-stained, abnormal
cytoskeletal elements, hyperphosphorylated tau protein
Lewy bodies: spherical, eosinophilic cytoplasmic inclusions
Viral inclusion bodies: Herpes virus group (HSV, CMV), Negri bodies
Storage Diseases: perikayal swelling with distinctive cytosomes
o Neuronal Necrosis: brightly eosinophilic (red) cytoplasm, pyknosis,
disintegration

o Neuron loss: surrounding (satellite) cells form nodules of Nageotte in DRG,


empty baskets of preterminal axons, microglial nodules
o Central chromatolysis: regenerative response
Large, rounded appearance w/ eccentric nucleus & Nissl substance
Pale pink central area of cytoplasm

o Axonal Changes: swelling at point of transection (spheroids) + Wallerian


degeneration distally

o Spongiform Degeneration: typical of prion DZ, small vacuoles in perikarya


Glia:
Return to Table of Contents

o Astrocytes
Fx: structural support/repair, BBB formation, isolation of neuronal
surfaces/NT metabolism, neural development
IHC: GFAP (can view nucleus and extensive processes)

Primary (injury to astrocyte): number and size increase w/o typical


reactive cytoplasmic changes
Secondary (parenchymal brain injury)
Increased glial filaments + glassy eosinophilic cytoplasm
plump cells called gemistocytes
Loss of reactive cytoplasm with time dense gliotic scar
May see bright red, irregular, rod-shaped bodies (Rosenthal)
o Oligodendrocytes
Fx: synthesis and maintenance of myelin sheaths around CNS axons
With LFB-PAS stain central myelin becomes bright blue
Small, round hyperchromatic nuclei with clear halo (fried egg)
o Ependymal cells
Fx: protective barrier and movement of CSF
Monolayer of ciliated epithelial cells lining ventricular system

Microglia/macrophages
o Fx: resident cells of monocyte/macrophage lineage
o Two routes of differentiation after activation by pathologic process:
Rod cell: found in aggregates (microglial nodule) a/w viral
encephalitis

Foamy macrophage: enlargement to spherical form w/ fluffy


cytoplasm

Return to Table of Contents

Vascular endothelial cells


Choroid plexus epithelial cells
o Cuboidal epithelium in ventricular system (primarily lateral & fourth
ventricles)
o Fx: produce CSF

o
Pituitary gland epithelial cells
Pineal parenchymal cells

Discuss the pathologic sequelae of the brain being soft and the pathologic lesions
that can arise from its packaging
Packaging:
o Cranium = cardboard box; prevent compression
o Meninges = Styrofoam; additional padding
o CSF = gel foam; innermost padding
Lesions arise in packaging
o Hematoma
Epidural: middle meningeal artery
Subdural: potential space in young, actual space in old; exposes
bridging veins that can shear with movement of dura
Subarachnoid: commonly seen in the inferior surface of brain
o Tumors (meningioma)
o Infections (meningitis)
Hydrocephalus (communicating and non-communicating) CSF plumbing defect
Traumatic Injury: impact against hard packaging, parenchymal tears w/
acceleration/deceleration
o Distribution: protuberant ends at front/back of brain

Return to Table of Contents

o
o When healed, residual bits of iron are present (hemosiderin)
Contrast communicating and non-communicating hydrocephalus
Communicating: flow of CSF is not blocked within the ventricular system
Non-communicating: flow of CSF is blocked along one or more of the narrow
pathways connecting the ventricles (often cerebral aqueduct)
Discuss the processes that occur in response to the presence of a space-occupying
lesion within the cranium
Monro-Kellie doctrine: V(brain) + V(CSF) + V(blood) [+ V(mass)]= V(cranium)
o Compensation redistribution of CSF or blood to accommodate mass
o Rapidly growing mass: quickly reach point of decompensation; ICP increases
o Slowly growing mass: brain itself can accommodate (through redistribution
of ECF) so decompensation occurs later on (larger mass required for ICP to
increase)
Herniation:

o
o Under falx cerebri
Return to Table of Contents

o Through tentorium cerebelli


o Through foramen magnum
Duret hemorrhage: secondary brainstem hemorrhage from herniation
o Bilaterally symmetric appearance (butterfly shape)
o Downward displacement of brainstem overlying blood vessels (basilar
artery) is fixed in place kinking/disruption of thin blood vessels

Describe the histologic changes that occur in response to ischemic neuronal death
Lesion distribution correlates with vascular territory
Early (hrs-days): edema, discoloration, hemorrhage
o Atrophic neurons w/ hypereosinophilic cytoplasm and dark, shrunken,
pyknotic nuclei
Intermediate (days-wks): soft material (remains soft when brain is fixed)
Late (months-yrs): macrophages remove dead material, leaving cavitary infarct
o Absence of neurons
o Reactive astrocytes/blood vessels at edge of cavity
o Foamy macrophages within cavity

Explain the consequences of the permanence of neurons


Accumulation of pigment (lipofuscin) represent protein and lipid-containing
residues of incomplete lysosomal digestion
o Pathologic inclusions are common in neurodegenerative diseases
Most degenerative disorders are neural since lost neurons are not replaced
o May have a paradoxically hypercellular appearance
o Astrocytosis can be extensive

o
Identify the pathologic process associated with focal cortical dysplasia and
hippocampal sclerosis
Seizure disorders: abnormally synchronous electrical activity which can spread
through the cortex
Focal cortical dysplasia cytologically abnormal area of cortex (starting point)
Hippocampal sclerosis (shrinkage and atrophy) unclear if cause or effect

Return to Table of Contents


List the two places within the brain that cerebral metastases and septic emboli
preferentially lodge and the reason for this finding
Gray-white junction: due to decreased caliber and increased tortuosity of vessels
Vascular border zones: decreased rate of blood flow in these areas
Contrast gray matter and white matter
Gray Matter

Pink when fresh, light brown when fixed


Lower cell density
Astrocytes/oligodendrocytes present
Cell bodies of neurons
Dendritic & axonal processes
Synapses
H&E: dark pink = cell bodies (pyramidal)
Lighter pink = neuropil EM: tangled
web of dendritic synapses/processes

White Matter

High, uniform cell density


Oligodendrocytes myelin sheaths
Astrocytes (fewer than
oligodendrocytes)
Projecting axons
H&E: largely free of neurons; small,
dark, round nuclei = oligodendrocytes

Identify manifestations of defects in neural tube formation


Rostral end defect = anencephaly (open neural tube)
Caudal end defect (more common) = myelomeningocele
Complete failure = craniorhachischisis totalis

Return to Table of Contents

10

Localizing Lesions in the Motor System

Paratonia/Gegenhalten: patient appears to hold or resist passive arm movements during


tone testing; may improve on distraction
Reflex Arc:

Alpha = innervate muscle fibers; Gamma = control spindle excitability


Ia fiber: fire when muscle spindles stretch muscle contraction

Ib fiber: fire when GTO over-stretch muscle relaxation


Return to Table of Contents

11

Lesions decreased muscle tone + hypo-reflexia

Discuss the motor signs and their pathophysiological mechanism in damage of:
Peripheral nerves/The lower motor neuron not discussed in lecture
A certain level of the spinal cord
o UMN pattern below the level of injury
o LMN pattern at the level of injury
o Contralateral dorsal column/medial lemniscus (light touch, vibration, joint
position)
o Ipsilateral lateral spinothalamic tract (crude touch, pain & temperature)
o Cauda Equina (spinal cord ends at L2) only LMN axons
o Key Dermatomes:
C4: clavicle
C8: fifth finger
T4: nipples
T10: umbilicus
L1: inguinal ligament
L3: anterior surface of thigh
L5: great toe
S1: lateral aspect of foot
S3-5: perineum
o Key Myotomes:
C3-5: Diaphragm
C5: deltoid, biceps (biceps reflex)
C7: triceps, extensors of wrist and fingers (triceps reflex)
C8: interossei, abductor of fifth finger
L2-4: quadriceps (knee jerk reflex)
L5: long extensor of great toe, anterior tibial
S1: plantar flexors, gastrocnemius (ankle jerk reflex)
The corticospinal tracts in the brainstem long tract signs
o Normal Function:
Suppress increased muscle tone and hyper-reflexia mediated by the
reflex arc
Generates movements: (1) stimulates LMN, (2) modulates interneurons, (3) modulates muscle tone
o Often associated with sensory deficit
o Hyper-reflexia
o Spreading of stretch reflexes (Hoffman sign, crossed adductor reflex)
o Spasticity velocity-dependent increased muscle tone
o Clasp-knife reaction resistance to passive movement increases up to certain
length/force sudden relaxation as GTOs are activated
o Pain-sensing reflexes are hyperactive Babinski sign, triple flexion response
o Clonus
Motor cortex
Basal ganglia

Return to Table of Contents

12

o Normal Function: facilitation of wanted movements, suppression of


unwanted movements, initiation/maintenance of movements

o Parkinson Disease loss of substantia nigra


Upregulation of D2 (loss of inhibition from SNc)
Downregulation of D1 (loss of excitation from SNc)
Overall increased inhibition of thalamus

Return to Table of Contents

13

Bradykinesia, rigidity, resting tremor, decrement on


repetitive/alternating movements, dystonia, NO weakness
o Ballism & Hemiballism loss of subthalamic nucleus
Inability to deselect undesired movements

o Huntington Disease loss of striatum


Early: loss of D2 inability to stop extraneous movements
Late: loss of D2 and D1 inability to choose desired movements

Return to Table of Contents

14

Cerebellum
o Ipsilateral deficits (overshoot/undershoot)
o Lateral cerebellum for limbs (limb ataxia/dysmetria and tremor)
o Medial cerebellum for trunk (ataxic gait, trunk ataxia/titubation)
o Hypotonia, but no loss of reflexes
Thalamus: many possible symptoms, but sensory symptoms predominate
Brainstem (important for neighborhood signs):
o LMN lesion of cranial nerves at level of lesion + UMN lesion below level of
lesion
o Vestibular nuclei/vestibulospinal: posture control w/ head movement
o Reticular nuclei/reticulospinal: somatosensory control of posture
o Superior colliculi/tectospinal: visual control of posture
o EX: facial nerve nucleus
Upper face has bilateral projections from cortex, lower face gets
contralateral projections only
Cortical lesions on one side:
Weak contralateral lower face
Sparing of contralateral upper face
Weak contralateral limbs
Brainstem lesions on one side gives crossed deficits:
Weak ipsilateral upper/lower face
Weak contralateral limbs

Describe the postural mechanisms and what happens when they are deprived of
higher control
Decorticate posturing (cerebral hemispheres, internal capsule, thalamus)
o Flexed upper extremities, extended lower extremitites
Decerebrate posturing
o Brainstem damage below level of red nucleus (mid-collicular)
Return to Table of Contents

15

o Dorsal root transection eliminates signs (gamma motor neurons and spindles
are involved)
o Extended upper and lower extremities
Both indicate severe brain damage
Progression from decorticatedecerebrate suggests further significant brainstem
injury or compression

Explain how the supplementary motor area, premotor area, and prefrontal cortex
contribute to the control of movement

Return to Table of Contents

16

Localizing Somatosensory Lesions


Contrast peripheral nerve fiber types
Larger myelinated system: vibration, position sense, touch
o Faster (good for temporal resolution)
o More precise (good for spatial resolution)
Small fiber system: nociception
o A-d: scantily myelinated
o C: unmyelinated
o Slow pain with diffuse character poorly localized
Identify patterns of sensory deficits associated with damage to peripheral nerve,
plexus, and spinal nerve root
Peripheral Nerve Disease
o Reflexes are decreased or absent
o Pattern analysis
Distal polyneuropathy: most common (stocking-glove pattern, longest
fibers affected first, combination of negative and positive symptoms)
Mononeuropathy (pressure, trauma, vasculitis) anatomy is key
Ex/ Carpal Tunnel
o Nerve conduction studies
Plexopathy (brachial, lumbar)
Radiculopathy (disk disease, shingles)
o Radiation of pain
o Segmental/dermatomal organization
Define referred pain
Visceral sensation loss of parallel processing due to absence of proprioceptors in
visceral organs
Pain in pattern of peripheral dermatome
o Cardiac pain (left arm and neck pain)
o Ureteral pain (groin/testicular region)
o Diaphragmatic pain (neck)
Therapy addresses root cause
Describe sensory deficits associated with lesions of spinal cord, thalamus, and
cerebral cortex
Dorsal column medial lemniscus system
o Uncrossed in the cord
o Synapse & decussation at medullary level
o Project to thalamus
o Clinical testing vibration & proprioception
Spinothalamic system
o Crossed path (mostly)
o Anterolateral portion of cord
o Projects to thalamus, reticular substance, tectum
Return to Table of Contents

17

o Clinical testing: sharp (pain), temperature


Typical Findings of Spinal Cord Lesion
o UMN findings: spasticity, increased tone, hyperreflexia
o LMN findings at level of the lesion
o Bladder dysfunction
o Find the sensory level to localize lesion
Syringomyelia cavitation of the central spinal cord (most often cervical)
o Cape distribution of pain/temp loss
o Sometimes segmental atrophy
Brown Sequard Syndrome (trauma or demyelinating lesion)
o Contralateral pain/temperature loss
o Ipsilateral proprioception/vibration loss
Thalamic Sensory Syndromes (from strokes, tumors)
o Negative signs include all modalities (highest level where this occurs)
o Full hemisensory deficit common
o Ventrobasal thalamus relays to SS cortex
o Intralaminar nuclei and central lateral thalamus involved in pain system
Cortical Sensory System
o Components
SI (postcentral gyrus) somatotopic organization; all contralateral
SII some bilateral input, less well-known function
Parietal association areas
o Characteristics
Cortical organization reflects the use of function
Plasticity exists for cortical organization
o Positive symptoms generated by seizures
o Testing function 2-point discrimination, graphesthesia, stereognosis

Discuss treatment approaches to pain associated with peripheral and central causes
Peripheral Neuropathy Pain
o Optimize nutrition, protect feet, reassure
o NSAIDs are of limited value
o Anticonvulsants: stabilize neurons that can fire spontaneously
Gabapentin, pregabalin (more commonly used)
Pregabalin binds to subunit of calcium channel modulates
calcium influx, reduces NT release
Pharmacologic effect requires binding at this site
Topiramate
o Antidepressants: combination of serotonergic and adrenergic reuptake
blockers
Amitriptyline, desipramine, duloxetine
Diminish the incoming pain signal via descending inhibitory pathways
o Topical therapy (capsaicin)
Capsaicin active ingredient in chili peppers
VR1: responds to heat stimuli, pain, acidic environment
Desensitizes receptors
Return to Table of Contents

18

o Opioids
Carpal Tunnel
o Reduce edema
Diuretics
Stop NSAIDs stop sodium retention
Treat hypothyroid myxedema
o Immobilize wrist splints
o Surgical release
Radiculopathies
o Reduce inflammation (NSAID, corticosteroid)
o Surgical decompression if major functional compromise or chronic pain
o Neuropathic pain therapies (as in peripheral)
Central Pain (cord lesions, thalamic pain syndromes)
o Regulation of sodium channels may play significant role
o Anticonvulsants
o Antidepressants (augment the descending serotonin pathways)
o Implanted stimulators
o Surgical lesioning
o TLR-4 antagonists
Inhibit the release of neuroexcitatory compounds and
proinflammatory products by glia
Opioids activate glia (oppose analgesia; enhance tolerance,
dependence, reward, respiratory depression)

Return to Table of Contents

19

Spinal Disorders
Describe the symptoms and signs of radiculopathy vs. myelopathy
Myelopathy: compression of spinal cord
o Dysfunction of spinal cords ascending and descending tracts
o Motor: Difficulty with fine motor fx weakness of multiple motor groups
spastic paralysis (does not occur in a single nerve root distribution)
o Sensory: Tingling diffuse numbness (does not occur in single root
distribution)
o Reflexes: hyper-reflexia, Hoffmans sign, Babinskis sign
Radiculopathy: compression of nerve roots
o Motor: weakness of single motor groups
o Sensory: numbness, tingling in a dermatomal pattern
o Reflexes: diminished isolated reflexes
Describe the basic anatomy of the vertebral column and spinal cord
Vertebral anatomy

o
o Differences between cervical, thoracic, lumbar vertebrae
Highest range of motion in cervical spine

o Unique aspects of C1 & C2 provides majority of rotation for head


Ligamentous anatomy
o C1-2 ligaments transverse ligament, right/left alar ligaments
o Other important spinal ligaments anterior and posterior longitudinal
ligaments, ligamentum flavum
Spinal cord anatomy
o Dorsal and ventral roots
o Differences between cervical thoracic, lumbar, sacral cord

Return to Table of Contents

20

o Tracts of the cord


Vascular anatomy: ASA (2/3) and PSA (1/3)

Identify the location of the major tracts within the spinal cord (corticospinal,
spinothalamic, dorsal column) and the deficits expected to result from interruption
of each of them

Corticospinal tract: main voluntary motor


o 90% crossed (at pyramids brainstem) via lateral pathway
o 10% - axial musculature uncrossed via anterior pathway
o Unilateral lesion in pons contralateral loss of movement
o Unilateral lesion in medulla cruciate paralysis
o Unilateral lesion in cord ipsilateral loss of movement
Posterior/dorsal columns: proprioception and light touch
o Crosses at level of medulla
o Unilateral lesion in cord ipsilateral loss of sensation to light touch
Spinothalamic tract: pain/temperature
o Crosses in spinal cord via anterior commissure (1-2 levels of entry)
o Unilateral lesion in pons/medulla/spinal cord contralateral loss

List the expected motor, sensory, and reflex abnormalities associated with
compression of each of the commonly involved cervical and lumbar nerve roots

Return to Table of Contents

21

Explain how the location of a herniated disk in the lumbar spine determines which
nerve root is compressed
Most commonly extrude postero-laterally compressing the nerve root in the
foramen radicular symptoms
Cervical disk herniations can extrude posteriorly to compress spinal cord
myelopathy
Level of disk herniation can be accurately predicted by neurological exam findings
o L4/5 and L5/S1 most commonly affected
Medial disk herniation compresses traversing nerve root (L4/5 L5)
Lateral/foraminal disk herniation compresses L4 (L4/5 herniation)
Describe how to approach the patient with a suspected spinal cord injury
Spinal trauma is common, should be suspected in high-energy trauma (car
accidents, falls more than 5, assaults)
Can be missed (1/3 of cervical injuries) associated intoxication, head injury,
multisystem injury
History + Physical Exam + Neuro Exam + Radiographic (lateral radiographs, flexionextension radiographs, CT, MRI)
Return to Table of Contents

22

Radiograph evaluate anterior & posterior longitudinal and spinolaminar line


Awake, cooperative, non-intoxicated patient
o No complaints of pain
o No pain to palpation of the spine
o Normal examination
o Low-risk mechanism
o Clinical clearance
Awake, cooperative, non-intoxicated patient
o No complaints of pain
o No pain to palpation of the spine
o Normal examination
o High-risk mechanism
o AP/lateral radiographs
Awake, cooperative, non-intoxicated patient
o Pain
o Normal examination
o AP/Lateral radiographs, CT, flexion-extension radiographs
Awake, cooperative, non-intoxicated patient
o Neurological deficit
o MRI (nerves), CT, flexion-extension radiographs
Intoxicated Patient
o No neurological deficit AP/Lateral radiographs, CT, wait until alert
o Neurological deficit MRI (neuro), CT, wait until alert
Diminished LOC
o Expected early return AP/lateral radiographs, CT, clear after normal LOC
o No expected early return AP/lateral radiographs, CT, MRI (24hrs), possible
flexion-extension under fluoroscopy

Describe the neurological manifestations of the common spinal cord injury


syndromes (Brown Sequard, anterior cord, posterior cord, central cord) and nontraumatic spinal cord syndromes
Brown Sequard hemisection of spinal cord (burst fractures)
o Ipsilateral motor loss below level of lesion
o Ipsilateral light touch loss below level of lesion
o Contralateral pain/temperature loss below level of lesion (1-2 levels lower)
Anterior Cord Syndrome (central disc herniations compressing ASA)
o Bilateral motor loss
o Preserved light touch
o Bilateral pain/temperature
Posterior Cord Syndrome (can be seen in fractures causing posterior compression)
o Preserved motor
o Bilateral loss of light touch
o Preserved pain/temperature
Central Cord Syndrome (trauma in setting of pre-existing stenosis)
o Motor loss: arms>legs; distal>proximal
o Variable light touch loss
Return to Table of Contents

23

o Variable pain/temperature loss


Secondary Injury
o Ischemia
o Free radicals
o Calcium cascade
o Endogenous opioids
o Inflammation
Tabes Dorsalis destruction of posterior roots at DREZ
o Tabetic neurosyphilis (15-20 yrs after initial infection)
o Lightning pains (most common in legs), ataxia (normal motor power),
urinary incontinence
o Absent LE reflexes, impaired vibration/position sense in feet, Romberg sign
o Normal MRI, positive VDRL
o Phenytoin/Tegretol (lightning pain), physical therapy
Subacute Combined Degeneration
o Lack of IF B12 deficiency
o Insidious onset: generalized mild weakness/paresthesia, gait disturbance
o Symmetric loss of positional and vibrational sense (limbs/trunk), variable
reflexes, associated mental signs
o Diffuse, uneven degeneration of white matter tracts + swelling of myelin
sheaths
ALS Lou Gehrigs disease
o Loss of nerve cells in anterior horns of spinal cord, motor nuclei of lower
brainstem
o Weakness/stiffness/awkwardness of hands
o Cramping/fasciculations of arms/shoulders
o Unilateralbilateral (painless)
o Mild spasticity in legs, hyperreflexia, muscle wasting in UE extensors,
preserved sensation
Transverse Myelitis
o Acute onset motor, sensory and/or autonomic dysfunction (no pre-existing
disease or spinal cord compression)
o Often multiple sclerosis (women>men; younger>older)
o Diagnosis w/ MRI, clinical suspicion

Return to Table of Contents

24

Neuromuscular Disease
Describe the important features of the clinical analysis, identification of the
anatomical site of disease, and diagnostic testing to be considered in the
neuromuscular evaluation
Gather data (history & exam) and organize data (describe clinical syndrome)
Process data localize lesion, differential diagnosis, labs, diagnosis, treatment
Anatomic Localization
o Arms vs. Legs vs. Cranial
Common legs early in disease
Unusual arms only at disease onset
o Proximal vs. Distal
Common: proximal (myopathy, neuronopathy), distal (axonal
neuropathy)
Uncommon: very proximal (respiratory failure, posterior neck
weakness)
o Symmetry
Asymmetric myopathies are rarely treatable
Asymmetric neuropathies are often treatable after nerve biopsy
o Focal regions shorter differential diagnosis, especially w/ specific weakness
Functional Patterns
o Motor: weakness (large vs. small muscle involved), abnormal movement
o Sensory: loss (small vs. large axons), gain (pain, paresthesia)
o Autonomic
Temporal Patterns
o Short vs. Long Term
Long: acute (days-wks), chronic (months-years), episodic, hereditary
Short: minutes-hrs, fatigue
o Onset Age
Pediatric: neonate & child
Adult: 20-60 y/o vs. >60 y/o
Potential Labs
o Serum biomarkers (i.e., creatine kinase, aldolase) indicate tissue involved
and disease activity
o Electrophysiology nerve conduction studies and electromyography
Nerve conduction: maximally stimulate nerve, record evoked action
potentials (muscle CMAP, sensory SNAP)
Ex/ CMAP small potential size axon loss/myopathy vs. slow velocity
myelin pathology
EMG: electrical activity detected by needle in muscle (motor units and
single muscle fibers)

Return to Table of Contents

25


Define the characteristic clinical, electrophysiologic, and laboratory findings in
muscle disease
Clinical Findings
o Weakness: proximal, constant
o Muscle size: early normal/increased late atrophy/increased
o Sensory exam & tendon reflexes are normal
Lab Findings
o Serum creatine kinase (CK): high
EMG: indicates small motor unit size
o Motor unit potentials have small amplitude and brief duration
o Single muscle fibers show spontaneous activity & occasional fibrillation

o
Muscle Biopsy
o Variable fiber size; small rounded shape (no compression to create polygon)
o Necrosis (phagocytic cells) + regeneration (basophilic w/ large, immature
nuclei) + increased endomysial connective tissue

Clinical example: Duchennes Dystrophy (absence of dystrophin w/ staining)


o Western blot can quantify dystrophin protein (milder Becker MD)
o Genetic testing (Southern blot) indicates out of frame deletion exon 45-48
o Pathogenesis: altered muscle membrane mechanical stability
o X-linked recessive (seen in males); progressive disease can involve heart
o Treatments: corticosteroids, manage dysfunction
o Future treatments: protein replacement, gene repair

Define the characteristic clinical, electrophysiologic, and laboratory findings in


disorders of the neuromuscular junction
Return to Table of Contents

26

Clinical Findings
o Weakness proximal, distal, bulbar
o Post-synaptic disorders (myasthenia gravis) decrement/fatigue
o Pre-synaptic disorders (Lambert-Eaton) increment w/ repetition
o Sensory & tendon reflexes are normal
o Chronic disease that rapidly changes with physical activity/treatment
Repetitive Nerve Stimulation
o MG: CMAP size decrement that improves w/ AChE inhibition

o
o LEMS: CMAP is small with increment after RNS or exercise; improves with
3,4-diaminopyridine (facilitates presynaptic vesicle release via Ca influx)

o
Laboratory Findings
o MG: serum antibodies to postsynaptic antigens (AChR 85%, MuSK 5%,
LRP4 5%)
Damage to post-synaptic membrane (simplified folds, wide cleft)
Decreased number +/- function of AChRs
o LEMS: serum antibodies to presynaptic antigen (P/Q calcium channel)
Reduced release of synaptic vesicles
Clinical Example: Myasthenia Gravis
o Focal weakness & fatigue: eyes, face, bulbar, arms>legs
o Limited eye movements improve after Tensilon treatment
o Treatment: pharmacologic (anti-AChE) and immunosuppression
o Look for thymoma (neoplasm association)
Clinical Example: Lambert-Eaton Myasthenic Syndrome
o Proximal weakness w/ sensory (distal, symmetric)& autonomic neuropathy
o Ataxia
o Treatment: 3,4 diaminopyridine & immunosuppression
o Look for small cell lung cancer, neoplasm association

Define the characteristic clinical, electrophysiologic, and laboratory findings in


diseases of nerve due to damage of the AXON
Clinical Findings
o Weakness: distal>proximal; asymmetric or symmetric; constant over time
o Atrophy of muscle size
o Distal (stocking-glove) pattern of sensory loss
o Reduced distal tendon reflexes (others are relatively spared)
Return to Table of Contents

27

o Symmetry is variable
EMG: indicates large motor unit size (sprouting from nearby neuron)
o Motor unit potentials have large amplitude, long duration
o Single muscle fibers show spontaneous activity, fibrillations

o
Nerve conduction testing: loss of axons w/ normal-mildly slow conduction velocity
o Loss of axons: small action potentials seen in CMAPs
o Normal velocity: remaining axons conduct w/ normal mildly slow velocity
Muscle biopsy
o Varied size; small fibers show angular appearance from denervation
o Denervation + re-innervation grouped atrophy w/ angular pattern
o Fiber type grouping demonstrates re-innervation (normal=checkerboard)
o Normal endomysial connective tissue

o
Nerve biopsy shows axonal loss (of small and/or large axons)

(sprouting)
Clinical example: Immune Microvasculopathy
o Asymmetric axonal neuropathy w/ sensory involvement & disability
o Nerve biopsy shows differential fascicular loss of axons w/ subperineurial
edema (patchy disease)
o Staining for C-5b9 complement component showed deposits on endoneurial
capillaries
o Treatment: immunomodulation (corticosteroids)

Define the characteristic clinical, electrophysiologic, and laboratory findings in


diseases of nerve due to damage of the MYELIN
Return to Table of Contents

28

Clinical findings
o Weakness: distal & proximal, constant, mostly symmetric
o Normal muscle size
o Distal, mild sensory loss
o Diffusely reduced tendon reflexes (out of proportion to weakness)
Nerve conduction testing
o Axons conduct abnormally conduction block with slow velocity
o Distal nerve stimulation gives normal evoked motor amplitudes
o Proximal nerve stimulation gives reduced evoked motor amplitudes
o Focal areas of myelination with some re-myelination (shorter internodes)
Nerve Biopsy
o Myelin sheaths show segmental loss with thin myelin sheets

o
o Schwann cell basal lamina onion bulb formations

o
Clinical Example: Guillain-Barr syndrome
o Acute onset with progression over 2-30 days after prodromal illness
Molecular mimicry (infectious agent + neural antigen)
o Weakness is diffuse (proximal & distal) & can be severe (respiratory failure)
o Early paresthesia + pain distal sensory loss, especially vibration
o Reduced tendon reflexes early in course
o Autonomic involvement
o Treatment: plasma exchange, IV immunoglobulin, NOT corticosteroids

Define the characteristic clinical, electrophysiologic, and laboratory findings in


diseases of nerve due to damage of the NEURON
Clinical Findings
o Selective modality loss motor or sensory, but rarely both
o Distribution: proximal + distal, cranial, not length dependent, asymmetric
o Motor: weakness, atrophy, fasciculations
o No improvement with treatment
Electrodiagnostic changes
o NCV: selective modality loss
o EMG: proximal and distal involvement that is not length dependent
Clinical Example: ALS
Return to Table of Contents

29

o
o
o
o
o

Weakness is proximal and distal, respiratory involvement, asymmetric


Variant involves no upper motor neuron signs
A4V mutation: rapidly progressive (death < 1yr)
Dominant inheritance (SOD1 gene mutation)
Accounts for 1-2% of all ALS

Return to Table of Contents

30

Neuropathology of Peripheral Nerve


Describe the structure and composition of peripheral nerves
Sural nerve: 6-12 individual fascicles of axons
Each fascicle surrounded by flattened cells of perineurium
Epineurium (outside perineurium) contains connective tissue and anastomic
vascular network (distributed to endoneurium)

Myelinated axons: 2-18 microns; myelin thickness related to axonal diameter


Unmyelinated axons are examined with electron micrograph
Skin Biopsies
o Allows examination of minute nerve endings in punch biopsies
o Limited to examination of small sensory terminal axons
o May show findings not found on sural nerve biopsy

Characterize the two processes in peripheral nerve pathology axonal degeneration


and segmental demyelination which are the pathologic substrates of most diseases
of peripheral nerve
Axonal Degeneration (often distal is preferred)

o
o Axonal degeneration starts w/ digestion of axon & myelin w/in Schwann cell
Return to Table of Contents

31

o Macrophages remove axonal and myelin debris, migrate to endoneurial


blood vessels and leave the nerve

o
o Schwann cell processes w/in original basal lamina band of Bungner
Collapse of basement membrane

o Though multiple axons begin to grow, one predominates


o After regeneration, internode length is shorter
Segmental Demyelination

o
o Intermodal length differs along the individual axon; some lack myelin

Return to Table of Contents

32

Correlate clinical patterns of peripheral nerve disease with their pathologic


substrates, identifying discrete classes of pathology (e.g., neuropathies arising from
ischemic, autoimmune, and metabolic insults)
Toxin-Induced Neuropathies (prefers distal aspects of nerves; lower extremities)
o Axonopathies (agents targeting axonal transport)
Effects of selected toxins or metabolic dysfunction on a phase of
axonal transport (may be selective)
Ex/ hexacarbons target neurofilament transport axonal swelling
Vinblastine, vincristine, taxol interrupt microtubule function (inhibit
axonal transport neuropathy)

o Myelinopathies
Lead, diphtheria toxin, hexachlorophene
Toxins directed at Schwann cell and/or myelin sheath w/ axonal
sparing
o Motor, sensory, and/or autonomic symptoms
o Acute, subacute, or chronic presentation
o Some involve selected functions
Ischemic Neuropathies
o Clinical Presentation
Development of sensory, motor or autonomic dysfunction (hrs - days)
Typically involves distribution of individual nerves (asymmetric)
Mononeuritis multiplex pattern hits on multiple nerves
o Vascular supply to peripheral nerve, if substantially decreased, disrupts fx
o Vasculitidies are patchy/focal thorough sampling of biopsied nerves
o Polyarteritis Nodosa prototypic vasculitic ischemic neuropathy
Epineurial arteries destroyed PMN leukocytes, macrophages,
monocytes, fibrin
Patchy pattern adjacent axons may be minimally involved
Fibrotic vessels w/ recanalization previous sites of damage

Return to Table of Contents

33

o Collagen Vascular diseases


o Other: paraneoplastic, cranial arteritis, AIDS, Lyme disease, diabetes, amyloid
& sickle cell disease
Metabolic Neuropathies
o Diabetes
Symmetrical sensori(motor) polyneuropathy
Variable degrees of axon loss in distalmost portions
Multiple ischemic hits summate, produce symmetrical and
uniform axon loss distally
Skin biopsies examine density and appearance of terminal
epidermal projections of sensory axons
Intraepidermal nerve fibers show large swellings (impaired
heat and pain thresholds)
Patchy loss of axons, thickened endoneurial microvasculature
due to failure to properly turnover glycated proteins

Asymmetric Neuropathies
Lumbosacral plexus neuropathy, truncal radiculopathy
Upper limb mononeuropathy, cranial nerve (III) palsies
May involve autoimmune component inflammatory vasculitis
Distal Autonomic Neuropathy (sympathetic, parasympathetic, visceral
sensory and enteric)
Markedly enlarged dystrophic nerve terminals w/
neurofilaments in the absence of substantial neuron loss

Return to Table of Contents

34

Autoimmune Neuropathies
o Guillain-Barre syndrome
Acute inflammatory demyelinating polyneuropathy (AIDP)
Monophasic paralytic illness (days-weeks after inciting event)
Rapid worsening paralysis & respiratory dependence
Sensory + autonomic dysfunction may be superimposed
Generally spares CNS functions
Improvement is the rule w/ residua common (mortality <5%)
Demyelination (wavy appearance) w/ preferential axonal
preservation; myelin debris around intact axon

Acute Motor Axonal Neuropathy (AMAN)


Targeted part is the axon, not the myelin
Campylobacter jejuni cross reacts w/ ganglioside GM1
Acute motor sensory axonal neuropathy (AMSAN)
Miller fisher variant
o Chronic inflammatory demyelinating polyradiculoneuropathy (CIDP)
o Experimental Allergic Neuritis (EAN)
o Anti-Myelin Associated Glycoprotein (MAG) Neuropathy
Anti-MAG IgMs bind to myelin sheaths of large & small axons
Produces biphasic myelin myelin sheath appears to be unraveling

o Anti-glycolipid, anti-sulfatide, anti-ganglioside neuropathies


Return to Table of Contents

35

Genetic Neuropathies
o Charcot-Marie-Tooth Neuropathy (PMP22 gene mutation)
History of motor weakness/ataxia
Atrophy, chiefly muscles of lower limb stork leg/inverted
champagne bottle
Lesser sensory and autonomic involvement
Family history typical, can extend over several generations
Onion-bulb (concentric rings of Schwann cell processes & collagen)
neuropathy with demyelinated axons

o Hereditary Neuropathy w/ Pressure Palsies (HNPP)


Dominantly inherited w/ deletions in PMP-22
Marked focal hypermyelination; redudant myelin folds, demyelination
& remyelination
Amyloid Neuropathies
o Predominantly sensory symptoms w/ painful burning extremities
o Lower extremities > upper extremities
o Autonomic symptoms common: postural hypotension, bowel/bladder
dysfunction, impotence, sweating abnormalities
o Endoneurial deposits of amyloid, often perivascular

o
Infectious Neuropathies
o Herpes Zoster
Latent virus in dorsal root ganglion emerges to give shingles
Neuronal and satellite cell intranuclear inclusions

Return to Table of Contents

36


o Leprosy
o AIDS
Distal sensory NP
Large numbers of macrophages in peripheral nerve

Toxic anti-retroviral drug rx NP


Mononeuritis multiplex NP (vasculitic/CMV)
Inflammatory demyelinating NP (Guillain-Barre, CIDP)
Opportunistic infectious (CMV polyradiculopathy, HZ radiculopathy)
Neoplastic (lymphomatous) NP
Diffuse infiltrative CD8+ lymphocytosis
o Lyme disease
Traumatic Neuropathy
o Loss of normal nerve continuity terminal swelling (mass of axons)

o
o Mini-fascicles of nerve can be seen w/ disorganized aggregate of collagen

Return to Table of Contents

37

Localizing Lesions of the Visual Pathways


List the principle eye muscle tested in each of the diagnostic gaze positions

Identify the eye movement findings associated with palsy of cranial nerves III, IV, VI

Abducens Palsy:
o Nuclear:
No esotropia
Paralysis of gaze toward the side of lesion; eyes remain aligned
Ipsilateral facial nerve paralysis w/ involvement of CNVII
o Subnuclear/Fascicular: brainstem syndromes w/ ipsilateral esotropia +
associated neighborhood signs
o Peripheral: esotropia (inward turning) of involved eye
Trochlear Palsy: eye is high and excyclotorted; vertical deviation is exaggerated
when eye moves into adduction
Oculomotor Palsy: affected eye deviates out and down + ptosis + mydriasis
o Pupillary fibers congregated toward outer edge ischemic lesions (DM/HTN)
affect deep fibers for lid/EOM pupil-sparing CN3 palsy
o Compressive lesions affect both pupils and EOM

Describe the pathways for conjugate horizontal gaze saccades and pursuits
Saccades
o Frontal eye field (FEF) paramedian pontine reticular formation (PPRF)
o PPRF VI and contralateral CN III (via medial longitudinal fasciulus)
CN VI nucleus is center for all horizontal eye movements except
vergence movements (convergence tied to accommodation)
Return to Table of Contents

38

Internuclear ophthalmoplegia (INO) lesion of MLF


Complete adduction deficit in severe cases
Adducton delay (all cases)
Abducting nystagmus in eye contralateral to adduction deficit
Young MS; old MLF stroke
VOR: acts entirely within brainstem tests integrity of brainstem pathways in
unconscious/comatose patients
Optkinetic nystagmus (OKN): steady entire visual world as it slips across retina
o Principal movement of eyes is in direction of slow phase (opposite direction
of object movement)
o Fast phase compensatory movement
Smooth pursuits
o Highly accurate; requires visual concentration and slowly moving target
o Open-Loop/Ballistic phase: faster than object movement to catch up to target
o Closed-Loop: lock on to target once caught-up
o Retina occipital lobe TPO junction cerebellum CN nuclei (via nucleus
prepositus hypoglossi)

Discuss the pupillary responses associated with the dorsal midbrain syndrome
Edinger-Westphal nucleus
Marcus-Gunn Pupil Afferent Pupillary Defect (APD)
o Decreased input into light reflex from one optic nerve
o Poorer constriction of both pupils when light is shone in involved eye
o Retina (massive destruction >70%)
o Optic nerve any lesion
o Optic chiasm if vision loss is asymmetric, APD in more affected eye
o Optic tract APD is contralateral to the lesion
Anisocoria efferent abnormality of the pupil (not related to APD)
o May be sympathetic or parasympathetic defect
o Is the anisocoria greater in the dark or the light?
Dark abnormal pupil is small, not dilating sympathetic
Light abnormal pupil is large, not constricting parasympathetic
Describe the pathway by which visual information is processed
Photoreceptors bipolar cells ganglion cells (optic nerve)
Optic nerve optic chiasm (nasal retinal fibers decussate) lateral geniculate
nucleus
LGN optic radiations (parietal & temporal lobes) primary visual cortex
Define visual acuity and how it is expressed
The eyes ability to distinguish details
Fraction: numerator test subject is standing at 20 feet; denominator distance
from which a normal subject can read the same image
Simple & quick, but does not help localize the lesion
Differentiate normal from abnormal visual field plots (perimetry)
Return to Table of Contents

39

Perimetry: focus on small target with lights of varying size/intensity presented in


various positions relative to fixation point

Describe the visual field deficits associated with a lesion of the eye, optic nerve, optic
chiasm, optic radiations, and occipital lobe
Sensory Eye Lesion
o Field defect in one eye only
o Follows boundaries of retinal nerve fibers
Optic Nerve Lesions
o Field defect in one eye only
o No anatomical boundaries for loss (central vision highly affected)

o
Optic Chiasm
o Damage to central chiasm affects nasal retinal fibers (temporal visual field)
o Bitemporal hemaniopsia

o
Optic Tract
o Right optic tract carries information from the left visual field of both eyes
o Complete contralateral homonymous hemianopsia

o
Optic Radiations
o Inferior portion (temporal lobe, Meyers loop) represents superior field
o Superior portion (parietal lobe) represents inferior field
o No anatomical demarcation between upper/lower; horizontal meridian is
not generally respected

o
Occipital Lobe
o Superior and inferior visual field halves separated by calcarine fissure
o Tip of occipital lobe (macula) dual blood supply: PCA & MCA

Return to Table of Contents

40

o
Pupils

Color

Return to Table of Contents

Acuity

41

Cerebral Blood Flow & Metabolism


Describe the effects of arterial pCO2 and arterial oxygen content on cerebral blood
flow
CBF responds to acute changes in arterial carbon dioxide
o Reduced pCO2 (hyperventilation) = reduced CBF
Can be used to reduce ICP (briefly)
W/ prolonged hyperventilation, CBF gradually increases
When pCO2 abruptly returns to normal, CBF rebounds to higher than
normal values (taper the return to normal)
o Increased pCO2 (COPD) = increased CBF
o These changes are lost when the brains vascular system is damaged (as in
ischemia) lose sensitivity to CO2
CBF responds to chronic arterial O2 content
o Reduced CaO2 (anemia) = increased CBF
o Increased CaO2 (polycythemia) = decreased CBF
Describe the effect of local neuronal activity on cerebral blood flow and metabolism
Normal: CBF matches CMR-O2 matches CMR-gluc
Increase in local neuronal activity
o Increase in local CBF
o Increase in CMR-gluc
o No increase in CMR-O1 activated brain relies on non-oxidative metabolism
of glucose
o Local cerebral venous oxygen content increases (not being used)
MR sequences sensitive to deoxyhemoglobin
fMRI: increased oxygen content = decreased venous deoxyhemoglobin
= increased signal intensity
Describe the regional relationship between cerebral blood flow and metabolism
Increased blood flow to active neurons
Corresponds to an increase in glucose metabolism, but NOT oxygen utilization
Illustrate the compensatory mechanisms used in the face of reduced cerebral
perfusion pressure
Autoregulation: changes in CPP (~60-150 mmHg) have little effect on CBF
o Systemic hypotension dilation of vessels
o Below threshold, vessels collapse (intramural pressure is too low)
o Chronic arterial hypertension shifts autoregulatory curve to right
The lower limit of autoregulation is higher than normal
Little change in limit is seen after 8-12 mo of treatment
Difficult to predict where the lower limit is
o This response is impaired when the cerebral vasculature is already dilated in
response to other vasodilatory stimuli
Anemia, hypoxemia, reduced CPP
Oxygen Extraction Fraction
Return to Table of Contents

42

o Brain normally extracts 30-40% of available oxygen


o When autoregulation fails, oxygen extraction increases
CBF drops
CMR-O2 is maintained
OEF increases
o When this compensatory mechanism fails (normal adults CBF
~20ml/100g/min), cerebral function declines and infarction may occur

Return to Table of Contents

43

Ischemic Stroke and Intracerebral Hemorrhage


Summarize the major risk factors/causes for ischemic stroke and intracerebral
hemorrhage (ICH)
Ischemic Stroke
o Age
o Gender: rate is higher in men (1.25), but women live longer, so they have
more strokes per year
o Race/ethnicity: AA, HA>white
o Hypertension
o Cardiac Disease
o Cigarette Smoking
o Diabetes Mellitus
o Hyperlipidemia
Pediatric Risk Factors
o Cardiac disease
o Heme disorder
o Vasculopathy
o Infection
Non-traumatic ICH
o Prolonged hypertension
Rupture of small arterioles
Basal ganglia, thalamus, cerebellum, pons, [lobar white matter]
o Abnormal arteries
Amyloid angiopathy rupture of small arterioles damaged by betaamyloid deposition
Predilection for lobar sites (especially posterior)
Occurs in older individuals, often in concert with dementia
o Clotting abnormalities
o Increased venous pressure
Describe the cerebrovascular changes that occur in the brain following ischemic
stroke
CT Imaging (Ischemic)
o Initially normal, subtle changes ~3 hrs in some (hyperdense vascular supply,
cortical effacement, loss of G-W differentiation, effacement of insular ribbon)
o Low density 12-24 hrs
MRI Imaging (Ischemic)
o Greater sensitivity than CT
o Increased signal on DWI, decreased on ADC within minutes
o Can estimate time of stroke onset based on findings on MRI sequences
In the first week: 10% get worse; 1% have recurrent stroke
CT (ICH): well-defined area of high density surrounded by rim of low density
MRI (ICH): determine approximate age of hematoma (esp T2* MRI)
Describe the causes of neurological deterioration after ischemic stroke and ICH
Return to Table of Contents

44

Ischemic stroke
o Brain factors
Cerebral edema can result in herniation syndrome
Bleeding into stroke
Progressive thrombosis
Stroke recurrence
Seizure
o Systemic Factors
Lungs: pneumonia, PE
Heart: heart failure, MI, arrhythmia
Metabolic/infectious: kidney failure, liver failure, fever/infection,
SIADH, drugs
o Hemorrhagic Transformation
Incidence within 5 days is 9%
Predictors: large lesion size, cardioembolic source, hyperglycemia,
thrombolytic treatment
Associated w/ poor outcome, only when exerts mass effect
ICH
o Hematoma Enlargement (usually within first 3 hrs): associated with decline
in consciousness when >33% growth
Factor VIIa reduces hematoma expansion, does not improve
functional outcome, death, or disability at 90 days

Illustrate the time course of recovery after ischemic stroke


Mortality after 1st Ischemic Stroke
o 5% in first 30 days ~5%/year
o Recurrence: 4% (30 days) 12% (1 year) 5% (per year thereafter)
Hemiplegia generally plateaus by about 3 months (improvement beyond this point
will be smaller)
Disability is common; at 6 months
o 50% hemiparesis
o 30% walk w/ assistance
o 26% dependent in ADLs
o 35% depression
o 26% institutionalized in nursing home
o At 5 years: ~50% disabled and dependent
Use location of infarct or ICH to predict likely underlying pathophysiology. Explain
limitations of this approach
Global/Multifocal Cerebral Ischemia
o Hypotension, hypoperfusion, low flow states
o Increased venous backpressure
o Diffuse vascular occlusion
o Functional vascular impairment
Cardioembolic Stroke (focal or multifocal)
o Atrial fibrillation/acute MI/ischemic or dilated cardiomyopathy
Return to Table of Contents

45

o Rheumatic valve disease/prosthetic valve/endocarditis/intracardiac tumor


o Abnormal passage from venous to arterial circulation
Borderzone Infarction
o Arterial vascular disease or systemic arterial hypotension
o Infarction in watershed areas between major cerebral arterial territories
Important Distinction: Carotid vs. Vertebrobasilar system
o Poor reliability of historical features
o Some deficits may occur with lesions in either system
Isolated motor or sensory deficits
Language deficits and neglect
o Anatomic variants
Variability in territory supplied by major cerebral arteries
Fetal PCA (originating from ICA rather than basilar artery)
ICA Territory (Anterior)
o MCA
Structure
Deficit
Primary motor area
Contralateral hemiparesis
(face/arm>leg)
Primary sensory areas
Contralateral sensory loss
(face/arm>leg)
Gaze pathways
Ipsilateral gaze deviation
Visual pathways
Contralateral visual field cut
Language center (left)
Aphasia
-nonfluent (superior MCA)
-fluent (inferior MCA)
-global (complete MCA)
Spatial attention center (right) Neglect
o ACA

Primary leg motor/sensory areas


Contralateral weakness +/- sensory loss of leg>>arm (face
spared)
Behavioral deficits, apraxia
o Akinetic mutism lack of verbal/motor responsiveness
o Abulia decrease in spontaneous speech & activity with
increased response latency, decreased persistence with
tasks
o Utilization behaviors difficulty resisting the impulse to
operate or manipulate objects that are in reach
o Apraxia loss of ability to carry out learned movement
not due to weakness, sensory loss, etc
o Ophthalmic Artery
Monocular blindness (transient: amaurosis fugax)
Vertebrobasilar Territory (posterior)
o Vertebral artery
o Basilar artery
Return to Table of Contents

46

PICA
AICA
SCA
PCA
Midbrain: hypersomnolence/cma, vertical gaze palsy, vivid visual
hallucinations (rare)
Thalamus: contralateral sensory loss/pain, contralateral ataxia,
aphasia (left), neglect (right), impaired
consciousness/cognition/memory
Occipital lobe: contralateral hemianopia, cortical blindness (bilateral),
visual association cortex involvement visual agnosia
Inferior temporal lobe: may have amnesia (left side)
o Brainstem: crossed or bilateral findings with cranial nerve abnormalities
and/or disorder of consciousness
o Cerebellum:
Hemisphere: ipsilateral dysmetria (+/- tremor), fall towards side of
lesion
Vermis: truncal/gait ataxia
Dyscoordinated speech and eye movements
Often associated brainstem signs
Lacunar Syndromes
o
o
o
o

o
Given the clinical scenario of a patient with ischemic stroke or ICH, discuss the most
appropriate evidence-based acute interventions and therapies to prevent
recurrence
Proven Benefit:
o IV tPA
Within 3 hrs of onset: 30% more likely to have minimal disability
Between 3-4.5 hrs: 16% more likely
Only 5-10% of patients receive this treatment (time window,
exclusion criteria)
o Clot retrieval (stentrevier)
More rapid recanalization + reduced hemorrhage risk
Risks: distal embolization, vessel injury, increased costs, delay in
treatment
MR CLEAN retrievable stent + IV tPA w/in 6 hrs of onset safe &
effective (in pts with proximal anterior circulation occlusion)
o Aspirin
o Low dose heparin
Return to Table of Contents

47

o Stroke unit admission


o Hemicraniectomy
For cerebral infarction with massive edema
Relieves pressure on midline structures
Early (<48hrs) for malignant MCA infarction reduces death (converts
deaths to those with moderate-severe disability)
Possible benefit: hyperosmolar therapy for edema
No benefit: heparin, neuroprotective agents (so far)
Unknown: BP treatment (may be harmful)
Secondary Stroke Prevention
o Antiplatelet therapy (15% RR)
Aspirin (thromboxane inhibitor)
Ticlopidine/clopidogrel (inhibitors of ADP-binding)
Aspirin/ERDP (phosphodiesterase inhibitor)
Clopidogrel, ASA/ERDP are slightly better than ASA, but more $$
Clopidogrel vs. ASA/ERDP similar efficacy; clopidogrel is
safer/better tolerated
o Antihypertensive therapy (28% RR) lowering just 9/4 mmHg
Indapamide (diuretic) & perindopril (ACE-inhibitor)
Benefit is seen even in people with normal starting BP
o Statins (16% RR)
Atorvastatin 80 mg decreases stroke risk when given at mean 3 mo
after stroke
For all individuals <75y/o w/ stroke due to atherosclerosis
(regardless of LDL)
o Endarterectomy for 70-99% ICA stenosis (50% RR)
o Oral anticoagulation for A-fib (66% RR)
Warfarin is better than aspirin
Dabigatran (direct thrombin inhibitor)
Rivaroxaban, apixaban, edoxaban (factor Xa inhibitors)
o Transfusion therapy for SCD (92% RR 1* prevention)
Must be continued indefinitely
o Smoking cessation has a presumed benefit
o Tight glucose control (DM), vitamin supplementation, stenting for
intracranial stenosis and bypass surgery for ICA occlusion have no benefit
Acute ICH
o Supportive care BP lowering is safe and may improve outcome
o Anticonvulsants if seizures (occur in ~15% of lobar ICH)
o Recombinant factor VIIa and surgery for deep hypertensive ICH have no clear
benefit (for cerebellar ICH, surgery may be useful)
o Reversal of anticoagulation and treatment of increased ICP have no proven
benefit
Secondary ICH Prevention
o Antihypertensive therapy
o Avoid anticoagulants if possible (especially lobar ICH)
o Treat specific cause
Return to Table of Contents

48

Compare the prognosis of ischemic stroke and ICH


Early Prognosis of ICH 1 month mortality is 50% (1/2 in first 2 days)
o Predictors of poor outcome: large ICH, low GCS, older age
o Care is withdrawn in 75% of patients who die in hospital (subject to
withdrawal of care biases self-fulfilling prophesy)
Long-term prognosis of ICH
o 6 months 50% of survivors are independent
o Recurrence rate 2-4%/year; mortality is 8%/year

Return to Table of Contents

49

Intracranial Pressure & Head Injury


Describe the major factors regulating intracranial pressure
Monro-Kelly Doctrine: volume (constant) = brain + blood + CSF
To a certain extent, the volumes of blood (venous) and CSF can compensate for
intracranial masses or for increasing brain size (edema)
Describe the primary compensatory mechanisms for increased intracranial pressure
Translocation of intracranial (venous) blood volume to extracranial vessels
Translocation of CSF to extracranial vessels via arachnoid villi
Translocation of CSF to spinal subarachnoid space
Molding of brain to a mass by deformation and/or reduction in ECF space
Brain shift/extrusion (herniation syndromes)
Contrast the pathophysiologic mechanisms, clinical manifestations, and radiographic
appearance of epidural vs. subdural hematoma
Epidural
o 1-3% of head trauma (M:F = 4:1)
o Pathophysiology: temporal skull fracture avulses middle meningeal artery
Arterial bleeding dissects between dura and inner table of skull
85% are arterial (as above), 15% from bleeding of dural sinus or
middle meningeal vein
o Clinical:
Classic: LOC + lucid interval deterioration w/ coma, contralateral
hemiparesis, ipsilateral pupil dilation
Mortality: 20-55%
o Imaging: biconvex (lenticular) hyperdense clot, constricted by suture lines

Subdural
o 24% of severe HI (75% male; 50% vehicular; avg age 41)
o Degree of impact damage is higher than in EDH (more lethal)
o Pathophysiology:
Laceration of bridging vessels (rapid acceleration-deceleration)
Accumulation due to rupture of parenchymal injury
o Clinical:
Often occurs with elderly patient who has fallen (on anticoagulant)
Mortality: 50-90%
o Imaging: concave/crescentic hyperdense clot, diffuse, crosses suture lines
Return to Table of Contents

50

May be along convexity, falx in interhemispheric fissure, or layering


on tentorium

Subacute: isodense clot


Chronic: hypodense clot

Discuss the complications of severe head injury


Monetary cost: $60 billion/year
Primary Damage: external forces acting to cause brain injury
o Macroscopic:
Shearing of WM tracts
Contusions
Intracranial hematomas
Diffuse swelling
o Microscopic/Cellular:
Rupture of vessels/microhemorrhages
Cell membrane microporation
Ion channel dysfunction
Protein changes
Secondary Damage: differs by 1* injury mechanism; processes last hrs-days
o NT release (glutamate): ion channel activation, swelling, edema
o Free-radical generation and Ca-mediated damage: early necrotic cell death
o Gene activation: pro-apoptotic proteins (caspases)
o Mitochondrial dysfunction: decreased ATP synthesis, cell death programs
o Inflammation
Poor Prognostic Indicators
o SBP < 90 mmHg; PaO2 < 60 mmHg; age > 40 y/o
o Elevated, uncontrolled ICP (sustained ICP >20-25 a/w 50% mortality)
o Low score on Glasgow Coma Scale (<8)

o
Glasgow Outcome Scale
o Death
Return to Table of Contents

51

o Persistent vegetative state


o Severe disability
o Moderate disability still independent
o Good recovery normal life +/- neuropsychological deficits
Severe TBI: poorer outcome, little improvement over time
Uncal Herniation: uncus of temporal lobe shifts into incisura
o Impairment of consciousness
o Dilation of ipsilateral pupil
o Ipsilateral or contralateral hemiparesis
o +/- decerebrate or decorticate posturing
Axial or Central Herniation: symmetrical shift of diencephalon & upper brainstem
into tentorial incisura
o Impairment of consciousness
o Dilation of pupils (usually bilateral)
o Decerebrate or decorticate posturing

Recognize concussion and the post-concussion syndrome


Concussion: traumatically-induced transient disturbance of brain function
o LOC, blindness, aphasia, loss of visual-spatial orientation, loss of situational
awareness, memory impairment
Post-Concussion Syndrome
o Headache, nausea, dizziness
o Impairment of concentration, learning disability, memory difficulties
o Symptoms usually resolve w/in days month
o Diagnosis/management complicated by mood disorders, ADD/ADHD, and
headache disorders
Management: cognitive & physical rest; neuropsychological testing for persistent
symptoms
Discuss guidelines for return to play after concussion
Premature return to cognitive or physical activity prolongs neurological dysfunction
Second injury before recovery further metabolic derangements + worsening of
cognitive deficits w/ possible long-term consequences

Return to Table of Contents

52

Subarachnoid Hemorrhage and Cerebral Vascular Malformations


Recognize the presenting clinical syndrome of subarachnoid hemorrhage
Thunderclap headache abrupt onset of severe headache
o Loss of consciousness may also occur
o Less severe character/diplopia may be a warning (sentinel) hemorrhage
Physical Exam Signs
o Nuchal rigidity
o Kernigs sign: pain in posterior aspect of upper legs when thigh is flexed and
knee is extended
o Brudzinskis sign: patients neck is flexed, involuntarily flex hips
o Third nerve palsy
Diagnosis
o Noncontrast CT if positive, usually dont need LP
o LP: sequential RBC count bloody fluid from hemorrhage or traumatic tap
Xanthochromia breakdown of hemoglobin leads to discoloration of
CSF (more specific for a subarachnoid hemorrhage)
o MRI not useful in making diagnosis
Describe the diagnostic evaluation of cerebral aneurysms
Vascular imaging: catheter cerebral angiogram or CT angiography
Cerebral Angiography gold standard
CT angiography 95% sensitive (vs. cerebral angiography)
o Advantage: non-invasive, can be obtained immediately after CT shows SAH
o Disadvantage: miss small aneurysms, bone artifact, highly operator
dependent
Describe management of aneurysmal subarachnoid hemorrhage
Surgical Management:
o Early surgery reduces risk of rebleeding
o Pterional craniotomy + operating microscope
o Spring-loaded clips separate aneurysm from circulation
o Morbidity/mortality is related to location (vertebrobasilar aneurysms >
anterior circulation) and size (max dimension > 25mm)
o Better long-term protection in patients < 40 y/o
Endovascular Management:
o Placing a coil into the aneurysm
o Increased chance of re-bleeding (small amount of aneurysm is left at base)
Peri-operative Management:
o Monitoring for and treating complications
o Anti-fibrolytic drugs keep clot in place to reduce re-bleeding rates
Discuss the complications of subarachnoid hemorrhage and how they are managed
Cerebral Vasospasm
o Progressive arterial narrowing (4-10 days post-hemorrhage)

Return to Table of Contents

53

May be due to oxyhemoglobin accumulating in subarachnoid space on


adventitial surface of the vessel
Loss of endothelial-dependent relaxation, inflammation, and free
oxygen radicals may contribute
o Can lead to cerebral ischemia
o Occurs in 1/3 of patients may have radiographic evidence without
symptoms; more likely in patients with thick subarachnoid clots
o Patients w/ poor neurological function at onset are at higher risk
o S/Sx: lethargy, fever, focal neurological deficits (hemiparesis, dysphasia)
o Treatment:
Calcium Channel Blockers (nimodipine)
Lessening severity of delayed cerebral ischemia
Can be associated w/ hypotension or cardiac dysrhythmia
Hemodynamic Therapy volume expansion to induce hypervolemia
(+/- pressors dopamine, phenylephrine)
Endovascular balloon catheter placement
Intra-arterial administration of verapamil
Hydrocephalus
o Occurs in 15% of patients more likely in pt w/ intraventricular hemorrhage
or severe subarachnoid hemorrhage
o S/Sx: new onset lethargy CT scan
o Management:
Initial: intraventricular catheter via frontal burr hole
Later: ventriculoperitoneal or ventriculoatrial shunt placement
Hyponatremia & other fluid/electrolyte problems
o Hypovolemia & total sodium depletion
o Do NOT treat with fluid restriction worsening of vasospasm
o Management: isotonic or hypertonic saline w/ monitoring of central filling
pressures

Discuss clinical presentation and management of cerebral arteriovenous


malformations
Direct shunting of arterial blood into cerebral venous system
Tendency to enlarge after formation in fetus mass lesion, seizure, ischemia
Symptoms occur in 2nd, 3rd, 4th decade of life
o Intracerebral/subarachnoid hemorrhage (occurs in 40-50%)
If present w/ bleed, more likely to re-bleed
o Seizures (~17-40%)
o Headaches or other neurological deficit
CT: enhancing mass of abnormal vessels
MRI: allows assessment of contiguity of functional brain structures
Grading: size, eloquence of adjacent brain, pattern of venous drainage
Treatment:
o Young pt w/ moderate size AVM in non-eloquent area of brain remove
o Microsurgical removal: craniotomy (after embolization therapy)
Pro: eliminate hemorrhage risk acutely
Return to Table of Contents

54

Con: invasive, surgical risk, prolonged recovery


o Stereotactic radiosurgery lesions < 3-4cm
Progressive obliteration of arterial and venous components
Disadvantage latency between administration of treatment and
effect of obliteration (up to 3 years)
o Embolization: less-invasive, but limited cure rate

Return to Table of Contents

55

Pathology of Circulatory Diseases of the CNS


Describe the anatomy of the CNS circulation
Internal Carotid System: ACA + MCA + short circumferential vessels (internal
capsule and basal ganglia) + paramedian vessels (base of brain)
Vertebro-Basilar System: PCA + SCA + AICA + PICA + short circumferential vessels +
small paramedian vessels
Discuss the most common etiologies of CNS vascular injury
Infarction (too little blood flow)
o Global Hypoxia
Ischemic Hypoxia (most common): cardiac arrest, hypotension,
marked increase in ICP
Hypoxic Hypoxia (reduced blood O2 content):
Reduced quantity of Hb: hemorrhage, severe anemia
Reduced oxygenation of Hb: respiratory arrest, status
epilepticus, carbon monoxide poisoning
Histotoxic Hypoxia (defective O2 utilization): cyanide poisoning
Hemorrhage (too much blood flow)
Atherosclerosis
o Develops at branch points & other sites of turbulent flow

o
o Narrowing of vasculature; thick, non-pliable vessel walls
Embolism
o Major Sources
Heart thrombus, endocarditis (infectious/non-infectious), calcific
material from aortic valve
Aorta & carotid arteries plaque rupture/thrombus
Peripheral vessels neoplasm, fat
o Shower of emboli from endocarditis scattering of small cavitations

o
Small Vessel Disease (arteriolosclerosis)
o Often seen in the white matter
Return to Table of Contents

56

o Concentric thickening (circumferential) of wall not irregular on one side


o Hyaline appearance pink & glassy w/ surrounding parenchymal retraction

o
o Pref. affects: basal ganglia, thalamus, pons, hemispheric white matter
Arteritis
o Giant Cell Arteritis fibrinoid necrosis of vessel wall

o
Watershed (Boundary Zone) adult and neonatal pattern
o Adults: ACA/MCA, MCA/PCA border zones
o Neonates: periventricular white matter (adjacent to edge of ventricle)
Venous (sagittal sinus & cortical vein thrombosis)
o Old: uncontrolled fevers/dehydration in children sluggish blood stasis
o Now: mostly caused by oral contraceptives

o
Hypertension
o Causes 50% of non-traumatic intracerebral hemorrhages
o Sites: putamen/thalamus (60%), cerebral hemispheres (20%), cerebellum
(13%), pons (7%)
o Most feared involve basal ganglia, pons, cerebellum

Return to Table of Contents

57

o
Subarachnoid Hemorrhage/Berry Aneurysm
o 90% in anterior circulation (ACA, ICA, MCA); 10% posterior

o
Cerebral Amyloid Angiopathy
o 12% of non-traumatic ICH; most common cause of lobar ICH
o Strongly associated with Alzheimers disease
o Carries considerable risk of bleeding (20%)

o
Vascular Malformations
o Irregular tangles of blood vessels with large caliber vessels in parenchyma
o Hybrid appearance of vascular channels arteries become veins w/o
intervening capillary plexus

o
Germinal Matrix (pediatric)
o Occurs in premature infants same distribution as watershed infarcts
o Delicate area around ventricle fine network of capillaries prone to
fluctuations in blood pressure

Correlate gross and microscopic features with the temporal evolution of an infarct
Return to Table of Contents

58

Gross Features
o Recent soft, discolored, pale (or hemorrhagic)

o
o Remote atrophy, cavitation, tract degeneration

o
Acute Hypoxia/Ischemic Damage (hours day)
o Hypereosinophilic cytoplasm (red and dead)
o Shrunken nuclei no visible nuclear substructure
o Microvacuoles may be present

o
o Selective Neuronal Vulnerability
Hippocampus: CA1, CA4 >> CA2
Neocortex: laminae III & V neurons
Cerebellum: Purkinje neurons

Subacute (day to week): macrophages, proliferative capillaries

Return to Table of Contents

59

o
Chronic (month year): macrophages, reactive astrocytes, cavitation

o
Discuss the gross and microscopic features of hemorrhagic lesions; given a
pathologic or radiographic image of an intracranial hemorrhage, identify the most
common etiology
Can occur w/ reperfusion of an area of infarction large bleed or many punctate
bleeds

o
Loss of blood vessel wall integrity spilling of RBC, WBC

Return to Table of Contents

60

Seizures and Epilepsy


Explain what seizure first aid is
Turn person on side; head inclined to ground keep airway clear; protect from
nearby hazards
Transfer to hospital if:
o Multiple seizures/status epilepticus
o Person is: pregnant, injured, diabetic
o New onset seizures
Do not put rigid object in mouth or restrain
Recognize the common presentations of seizures
Epileptic Aura
o Symptom at onset, precedes seizure by seconds minutes
o Implies partial onset of seizure
o Non-specific symptoms noticed for hrs days before seizure
Generalized Tonic-Clonic Seizures
o Last < 1 minute; 10-20 s tonic phase; often toxic or metabolic disorder
o Brief flexion
Muscles contract
Eyelids open, eyes look up
Arms elevated, abducted, externally rotated; semiflexed elbows
Legs less involved, may be flexed
o Extension phase more prolonged
First in back and neck
Cry may occur (2-12 seconds)
Arms extend
Legs extend, abduct, externally rotate
o Tremor
Repetitive relaxation of tonic contraction
8/s 4/s
Leads to clonic phase

o
Absence Epilepsy
o Brief (5-10 s) episodes of lapse of consciousness w/o aura or postictal
symptoms
o Onset: 4-8 y/o; typical spontaneous remission by mid-adolescence
o 3 Hz spike and wave pattern on EEG
o Treatment with AED is efficacious in 80-95%
Return to Table of Contents

61

Juvenile Myoclonic Epilepsy


o Onset: 12-18 y/o
o Myoclonic jerks occur mostly in morning; generalized tonic-clonic seizures in
90-95% of cases
o EEG: generalized polyspike and wave discharges
o Typically no spontaneous remission
o Control with AED in 90% of patients
Partial Seizures
o Begin in focal area of brain
o S/Sx vary according to site of onset
o Can be associated w/ underlying structural brain lesion
o Frontal Lobe
Jacksonian motor seizures primary motor cortex
May involve more complex motor symptoms
o Temporal Lobe
Associated with aura
Mesial Temporal Lobe epilepsy sclerosis of hippocampus and
surrounding structures
Onset: before/at time of puberty
Resistant to AED

Be able to classify seizure types and select an appropriate anti-epileptic drug (AED)
Generalized:
o Absence
o Myoclonic
o Tonic-clonic
o Tonic
o Clonic
o Atonic
Partial:
o Simple no altered consciousness
o Complex consciousness impaired
o Secondarily generalized
Seizure Type
Antiepileptic Drug
Broad spectrum (ALL TYPES)
Clobazam, felbamate, lamotrigine,
levetiracetam, rufinamide, topiramate,
valproate, zonisamide
Narrow: Partial seizures all subtypes
Carbamazepine, eslicarbazepine,
ezogabine, gabapentin, lacosamide,
oxcarbazepine, perampanel,
phenobarbital, phenytoin, pregabalin,
primidone, tiagabine, vigabatrin
Absence seizure
Ethosuximide
Know the common causes of seizures and how to diagnose them
Return to Table of Contents

62

DDx: seizure, syncope, migraine, cerebral ischemia, movement disorder, sleep


disorder, metabolic disturbance, psychiatric disturbance
Precipitants
o Low blood glucose (less often, high)
o Hyper and hypo-natremia
o Hyper and hypo-calcemia
o Hyper and hypo-magnesemia
o Stimulant/proconvulsant intoxication
o Sedative withdrawal
o Severe sleep deprivation

Know the indications for epilepsy surgery


Medically refractory epilepsy in adults
Discuss the role of EEG in the evaluation of seizures and epilepsy
EEG: measurement of electrical potentials from brain, recorded over scalp
o Interictal EEG
o Video EEG telemetry monitoring continuous synchronized video and EEG
recording; gold standard in making diagnosis
Used to clinically diagnose epilepsy
o Initial EEG: discharge in 29-55% of patients
o Serial EEG: 80-90% of patients
o Repeat studies + sleep deprivation + extended recording times
Normal: Posterior Rhythm
o 8-13 hz higher voltage over occipital areas
o Eyes closed, physical relaxation, relative mental inactivity
o Blocked/attenuated by attention (visual, mental effort)

o
Abnormal
o Focal supports partial seizure diagnosis
o Generalized supports generalized seizure diagnosis
Generalized poly-spike and wave discharge
Predominance over bi-frontal regions
Common interictal pattern in juvenile myoclonic epilepsy

Return to Table of Contents

63

Sleep Disorders
Describe normal sleep architecture
Qualitative and quantitative description of a persons sleep
o Onset & duration of sleep
o Consolidation of sleep, sleep fragmentation, wake time after sleep onset
(WASO)
o Time and cycling between different sleep stages
EEG-based, polysomnographic features
o EEG electrodes on scalp to evaluate brain wave activity
o Eye leads to identify REM sleep
o EMG to determine atonia (REM sleep), lower limb (PLMs)
o Air flow monitoring (nose + mouth) & snore microphone
o Chest & abdominal belts to assess respiratory effort
o Oximetry, CO2 monitoring
o ECG
Sleep Stages
o Awake posterior dominant rhythm

o N1 (stage 1, drowsiness)
Slow rolling eye movements
Low amplitude theta & delta
Alpha dropout
o N2 (stage 2, light sleep)
K complexes
Sleep spindles

o N3 (stage 3 & 4, slow wave sleep, deep sleep)


>50% is delta activity (1-3 Hz > 150mV)

o REM
Atonia, low chin EMG activity
Heart and respiratory variability
Low amplitude, high frequency EEG activity

o Cycles: occurs ~90 min intervals


Return to Table of Contents

64

Provide a differential diagnosis of excessive daytime sleepiness and discuss how to


evaluate a patient presenting with this complaint
Definition: drowsiness or sleep onset that occurs during inappropriate or
undesirable times
Evaluation: Epworth sleepiness scale (score >10/24 = abnormally sleepy)
EPWORTH SLEEPINESS SCALE - Please estimate your risk of falling asleep in the following situations, using the scale below.
SITUATION

Sitting and Reading


0= None
Watching TV
0= None
Sitting inactive in a public place (e.g. theatre or meeting)
0= None
As a passenger in a car for an hour without a break
0= None
Lying down to rest in the afternoon when circumstances permit
0= None
Sitting and talking to someone
0= None
Sitting quietly after a lunch without alcohol
0= None
In a car, while stopped for a few minutes in traffic
0= None
MEDICATION
OR
NON-MEDICATION
ALLERGIES
Differentiate from fatigue or tiredness

CHANCE OF DOZING

1 =Slight
1 =Slight
1 =Slight
1 =Slight
1 =Slight
1 =Slight
1 =Slight
1 =Slight

2= Moderate 3=High
2= Moderate 3=High
2= Moderate 3=High
2= Moderate 3=High
2= Moderate 3=High
2= Moderate 3=High
2= Moderate 3=High
2= Moderate 3=High

o
o
Possible Etiologies
o Inadequate sleep time
o Sleep fragmentation (sleep apnea, sleep related movement disorder RLS,
neurological or medical disorders, medications)
o Primary sleep disorders e.g., narcolepsy
o Circadian disorders

o Insomnia
Consequences
o Cognitive: impaired function, judgment, recall and response time
Return to Table of Contents

65

o Metabolic: impaired immune/hormonal function, reduced glucose utilization,


reduced physical endurance & increased perceived effort during exercise
o Each hour of wakefulness after 10 hrs = 0.004% rise in BAC (24 hrs = 0.1)
Describe the clinical presentation, health impact, and treatment options for patients
with obstructive sleep apnea
Disorder characterized by repetitive episodes of complete or partial upper airway
occlusion during sleep
o Sleep symptoms: habitual loud snoring, witnessed apneas
Snorting, gasping, nocturnal awakenings, nocturia
GERD, dry mouth, night sweats, difficulty sleeping supine
o Waking symptoms: excessive daytime sleepiness, fatigue
Morning headache, neuropsychiatric/mood changes
Sore throat, optic neuropathy, hearing loss
Diagnosis: five or more score-able respiratory events (apnea, hypopnea, RERA) per
hour of sleep on polysomnography
Health Impact (5% of population, higher in hospital, more common in men)
o Oxygen desaturation, arousal/awakenings at termination of event
o Cardiovascular Effects: HTN, CV disease, CAD, HF
o or more patients with acute stroke or TIA have sleep disordered breathing
Treatment
o Positive airway pressure CPAP, BIPAP
o Mandibular advancement devices
o Positional therapy
o Surgical treatment in selected cases
Relieve nasopharyngeal obstruction
Adenotonsillectomy (especially in kids)
Uvulopalatopharyngoplasty (UPPP)
Tracheostomy
Bariatric surgery
Discuss the presenting symptoms of narcolepsy and standard treatment for
excessive daytime sleepiness and cataplexy in a narcolepsy patient
Clinical Tetrad
o Cataplexy (pathognomonic for narcolepsy)
Muscle weakness; can be triggered by emotion
Consciousness preserved, eyes closed
Hypotonic, areflexic REM atonia
o Hypersomnolence
o Hallucinations (hypnogogic going to sleep/hyponopompic waking)
o Sleep paralysis
Sleep fragmentation, weight gain, depression/OCD
Diagnosis supported by negative PSG and MSLT: sleep onset REM episodes in > 2
nap tests
Treatment
o Stimulants for excessive sleepiness
Return to Table of Contents

66

Provigil (modafinil)
Amphetamines
Methylphenidate
o Tricyclics for cataplexy amitriptyline
o Sodium oxybate (GHB, Xyrem)
Extremely short half life take at night
Unclear mechanism
Discuss the clinical presentation and treatment options for patients with restless leg
syndrome and periodic limb movement disorder
Periodic Limb Movements
o 0.5-5 s burst of limb movement (EMG amplitude >25% of baseline)
o Intervals of > 5 s (usually 20-40 s, up to 90s)
o Repetitive, at least 4
o Associated w: sleep disordered breathing, narcolepsy, medications, ADD,
pregnancy, secondary disease (renal failure, neuropathy, iron deficiency, low
ferritin, Parkinsons)
o Diagnosis: sleep disturbance w/ daytime symptoms, PLM index >5/hr
(children) and 15/hr (adults)
Restless Leg Syndrome
o Mostly idiopathic, estimated 5% of general population
o May emerge in pregnancy, then resolve after
o Many cases autosomal dominant w/ high penetrance
o Secondary neuropathy, renal disease, spinal cord lesions, radiculopathy
o Clinical Diagnosis (w/o definitive objective diagnostic tests)
Irresistible desire to move legs, a/w paresthesias
Discomfort precipitated by rest, relieved by movement
Circadian rhythm of severity worse near habitual bedtime
Abnormal motor activity PLMs on PSG
Low serum ferritin treatment of this can improve symptoms
o Associated w: medical disorders (iron deficiency, uremia, connective tissue
disorder, neuropathy, myelopathy), precipitating factors (methylxanthines,
lithium, dopamine antagonists, antidepressants, iron deficiency)
o Nonpharmacological treatments: heat, massage, sleep hygiene
o Consider: painful legs & moving toes syndrome, nocturnal cramps,
neuropathy, akathesia, chorea
o Treatment
Dopaminergic agents pathophysiology-defective DA receptors
Benzodiazepines consolidate sleep
Opioids refractory cases
Antiepileptics (Gabapentin)
Secondary: adrenergic agents (clonidine, propranolol), baclofen,
quinine, others
Provide a differential diagnosis of insomnia
Adjustment Insomnia (Acute)
Return to Table of Contents

67

o Identifiable stressor, resolves with adaptation


o Symptoms last < 3 months
Psychophysiologic Insomnia (Learned)
o Heightened arousal and associations lead to insomnia
o Symptoms > 1 month
Paradoxical Insomnia (Sleep State Misperception)
o Reports no sleep, objective measures reveal more sleep than reported
o Constant rumination or awareness of environment through night
o Symptoms > 1 month
Idiopathic Insomnia
o Present since infancy or early childhood; no precipitant
o Persist without periods of remission
Insomnia Mental Disorder
o DSM for mental disorders; greater than expected for mental disorder
o Symptoms > 1 month
Inadequate sleep hygiene (>1 month)
Due to Drug/Substance (>1 month)
Medical Condition (>1 month)

Describe good sleep hygiene practices


Sleep restriction, stimulus control therapy
Describe general principles of treatment of insomnia with hypnotic medications
Cognitive behavioral therapy is most effective
Good sleep schedule and sleep habits are very important
Medications are helpful as adjuncts
Most commonly prescribed are antidepressants; only 4/16 top mentions are FDA
approved for insomnia
o Nonbenzodiazepine-selective benzodiazepine receptor agonists
o Zolpidem, eszopiclone, zaleplon (BZ1 receptor)

Return to Table of Contents

68

Coma, Persistent Vegetative State, & Death by Neuro Criteria


List the general types of pathologic processes that can impair consciousness
Depression of bi-hemispheric function
o Supratentorial mass lesion(s)
o Diffuse brain disease
o Metabolic disorders
Depression or destruction of reticular activating system (RAS)
o Infratentorial destructive/mass lesions
o Compression by supratentorial process
o Metabolic disorders (late)
Non-dominant fronto-parietal cortex lesion
Given pupillary or eye movement findings in a comatose patient, identify the level of
nervous system damage
Pupillary Size/Reactivity
o Bihemispheric /metabolic
Small, reactive pupils
Pupillary light reflex is resistant to metabolic damage
o Hypothalamic dysfunction (diencephalic):
Interrupts descending sympathetic pathways
Bilaterally small pupils
Light reflex preserved
o Midbrain dysfunction:
Interruption of light reflex (dorsal lesions - Tectum)
May see hippus spontaneous oscillation in pupil size that is
independent of illumination
Interruption of parasympathetic & sympathetic (nuclear/Tegmentum
lesions)
Midposition (may be large in tectal), fixed pupils
o Pontine Lesions:
Interrupts descending sympathetic fibers and ascending inhibitory
pupil dilator pathways
Bilaterally small (pinpoint) pupils
Light reflex preserved
o Peripheral CN III lesions:
Unilateral pupillary dilation and unreactivity
o CheyneStokes respiration:
Pupillary dilation during hyperpnea
Constriction during apnea (cervical sympathetic nerves)

Return to Table of Contents

69

o
Eye Movement Findings
o Dysconjugate (asymmetric) eye movement abnormalities structural
o Review Motor Pathway (looking to right):
Left FEF internal capsule/cerebral peduncle decussate in upper
pons right paramedian pontine reticular formation (PPRF)
Right PPRF right CN VI nucleus
Right CN VI nucleus MLF left CN III nucleus
o Oculocephalic (Dolls): turning head, stimulate proprioception
Positive conjugate eye deviation in opposite direction
Bifrontal/diffuse hemispheric disease (FEF damage)
Intact brainstem oculomotor pathways
Absent turn in same direction as head lower brainstem
Dysconjugate = upper brainstem lesion
o Oculovestibular Response: stronger stimulus than oculocephalic
Normal response = intact brainstem (presence/absence of slow phase,
not the fast phase)
Ice water in one ear- normal = slow deviation toward irrigated ear,
quick beating away as supratentorial FEF re-fixes vision
Supratentorial (bihemispheric/metabolic) = slow phase, no fast phase

Medial midbrain/pontine (MLF) dysconjugate gaze


Return to Table of Contents

70

Failure of adduction of eye ipsilateral to lesion

Mid-lower pontine lesion (PPRF) no response

Describe and interpret motor response in a comatose patient


Motor response to noxious stimuli
Lateralized deficit posturing on one side, purposeful (hemispheric) on other side
Posturing
o Decorticate: increased tone in UE flexors, LE extensors
o Decerebrate: increased tone in extensors of UE & LE
o Localization is not as precise in humans
o In general, decorticate has better prognosis
Better to report what happens with each limb (as these can be fragmented in
presentation and change over time)
Identify herniation syndromes based on clinical and radiographic features
Normal Radiographs:

o
o Left: midbrain level; uncus is lateral to suprasellar cistern
Central Herniation
Return to Table of Contents

71

o
o Progresses rostro-caudally through 4 stages (know general idea)
Pupils
Eye Movement Motor Res
Other
Dienceph Small
Full
Appropriate
Chance for
alic
or posturing
recovery
Midbrain Midposition, May be
Posturing or
DI, fluctuating
fixed
dysconjugate
none
temp
Pontine
Midposition, None
Posturing or
fixed
none
Medullary Midposition, None
None
Always
fixed, may
progresses to
widely dilate
death
Uncal Herniation

o
o Kernohans notch indentation in tentorium by contralateral cerebral
peduncle as uncus pushes midbrain to other side
Ipsilateral hemiparesis as midbrain pushed against opposite tentorial
edge
False localization sign
o Ipsilateral dilating pupil = earliest consistent sign (rapid progression after)
o Contralateral hemiparesis compression of ipsilateral corticospinal tract
o Compression of PCA unilateral/bilateral occipital lobe infarction
o Beyond this, indistinct from central herniation
Tonsillar Herniation

Return to Table of Contents

72

o
o Usually posterior fossa mass lesion
o Cerebellar tonsils herniate downward through foramen magnum
o Compression of medulla depression of respiratory centers and cardiac
rhythm control centers
o Sudden cardiorespiratory arrest (without being comatose first)
Differentiate coma due to metabolic disease from that due to a structural brain
lesion
Clinical Signs of Metabolic Coma
o Change in LOC or cognition
o Respiratory changes nonspecific
o Pupils remain reactive (w/ exceptions)
o VOR: normal or lack of nystagmus (slow phase intact, fast is lost)
o Motor abnormalities: tremor, asterixis, multifocal myoclonus
o Late: braintem signs
Differentiate psychogenic unresponsiveness from organic coma
Uncommon; usually w/ psychiatric history
Requires neurological signs and symptoms are anatomically/physiologically
impossible
o Lack of attention to environment
o Eyes closed: resist lid opening, then flutter upward (see sclera Bells
phenomenon), close rapidly when released
o Pupils normal
o +/- OCR, but VOR is normal (cannot be suppressed)
o No roving eye movements (cannot be mimicked would need to follow an
object to attain smooth movements)
o Respirations normal/hyperventilation
o No withdrawal from noxious stimuli; may avoid self-injury
May be due to conversion reaction, catatonic stupor, severe psychotic reaction, or
malingering
List the requirements for death by neurological criteria
Clinical Definition
o Irreversibility establish cause; exclude known reversible conditions, period
of observation (varies)
Return to Table of Contents

73

o No confounding conditions
o Absence of brain function + apnea on 2 exams
Unresponsive, absence of brainstem reflexes (pupils, OCR/VOR,
corneal, gag, cough)
Pupils midposition or dilated & fixed
Apnea- lack of significant respiratory effort
Requires potent stimulus (PaCO2 of 60-arbitrary)
Avoid hemodynamic instability
Spinal reflexes may be present
Additional testing is not required (may be of value, but cannot assure cessation)
o Radionuclide angiography
o EEG
o Cerebral angiography
o Transcranial Doppler

Return to Table of Contents

74

Pathology of CNS Tumors


Discuss the typical clinical, radiographic, and pathologic manifestations for each of
the common CNS neoplasms of childhood and adulthood
Astrocytic Tumors
o Pilocytic (grade I)
Clinical:
Child/young adult; Insidious onset/slow growth; surgically
curable (80% 20yr survival)
Cerebellum, hypothalamus/3rd ventricle, optic pathway, spinal
cord
Radiographic: Enhancing nodule cyst/mural

Pathologic:
Gross: sharp border between tumor and normal tissue
Histo: piloid (hair-like) processes

Biphasic growth pattern: dense area (Rosenthal fibers) + loose


microcystic area (eosinophilic granular bodies)

o Diffuse (grade II)


Return to Table of Contents

75

Clinical:
Age 30-40, insidious/slow-growing
Frequent progression to grades III or IV (survival 5-8 yrs)
Radiographic: non-enhancing + hyperdense on FLAIR

Pathologic:
Gross: blurring of G-W junction, discoloration of white matter
Histo: Atypia

o
o Anaplastic (grade III)
Clinical:
Age 40-50, rapid onset
Frequent progression to grade IV (survival 2-3 yrs)
Radiographic: some enhancement
Pathologic:
Atypia + Mitoses

o Glioblastoma (grade IV)


Clinical:
Return to Table of Contents

76

Age 50-60, rapid onset and progression


Survival 1- year
Radiographic: Ring-enhancing

Pathologic:
Gross central necrosis, peripheral hyperemia
Atypia + Mitoses + endothelial hyperplasia OR necrosis
Irregular, multilayer vessels multiple lumens (glomeruloid)

Nuclear pseudopalisading around zones of necrosis

Oligodendroglioma (Grade II or III)


o Clinical:
Age 30-40 years, corticotropism/seizures
Return to Table of Contents

77

10-20 yr survival grade II; 5-10 yrs grade III


Chromosome 1p & 19q co-deletions = enhanced survival &
therapeutic sensitivities (procarbazine, CCNU, vincristine)
o Radiographic:

Anaplastic (Grade III)


o Pathologic:
Gross: no G-W distinction
Uniformly round nuclei w/ perinuclear clearing, fine chromatin

Branching capillary network gives chicken-wire appearance

Ependymoma (Grade II or III)


o Clinical:
Kids (4th ventricle, supratentorial); adults (spinal cord)
Prognostic variables
Return to Table of Contents

78

Extent of resection/location (most important)


Patient age (poor if <2 y/o)
Histologic grade (anaplastic if grade III)
CSF dissemination (<5%)
o Radiographic: homogenously enhancing

o Pathologic:
Perivascular pseudorosette (vessel + nuclear free zone)

Ganglioglioma (Grade I)
o Clinical:
Children/young adults, chronic seizure disorder (temporal lobe)
Benign/surgically curable
o Radiograph: cyst/enhancing mural nodule w/ contrast

o Pathologic:
Binucleate ganglion cell, eosinophilic granular bodies, perivascular
lymphocytes

Return to Table of Contents

79

Medulloblastoma (Grade IV)


o Clinical
Children/young adults, aggressive natural history
CSF seeding (icing and drop metastases)
5 yr survival = 60-80% w/ therapy (medulloblastoma), 30% (PNETs)
Radiation risk-benefit analysis
Favorable = nodular/desmoplastic
Unfavorable = anaplastic/large cell
o Radiographic: homogenous enhancement

o Pathologic
Primitive small blue cell tumor (medulloblast)
Focal neuroblastic or Homer-Wright rosettes (central neuropil)

Meningothelial Neoplasms
o Meningioma (Grade I) most common extra-axial CNS tumor
Clinical:
Adults, F:M ratio =2; insidious/incidental
Return to Table of Contents

80

NF2 gene in sporadic and familial cases


5-yr recurrence 5% for gross total resection, 40-60% for
subtotal resection or high grade (II or III)
Radiographic: homogeneously enhancing, discrete

Pathology:
Laminated calcifications (psammoma bodies), whorling
pattern (cell wraps around itself), hyperostosis (thickening
from skull invasion)

o Atypical Meningioma (Grade II)


15-20% of meningiomas, high risk of recurrence
High mitotic index or brain invasion (radiation therapy)
o Anaplastic Meningioma (Grade III)
1-2% of meningiomas
Frank anaplasia (carcinoma, sarcoma, or melanoma-like)
More than 20 mitoses/10 HPF; high mortality
Effective adjuvant therapy is lacking
Sellar Tumors
o Pituitary Adenoma (adults)
Sheet-like or nested growth pattern, monomorphic

Return to Table of Contents

81


o Craniopharyngioma (children)
Frequently recur; morbidity is common (visual loss,
panhypopituitarism)
Microcystic spaces, basal nuclear palisading, stellate reticulin
(cobweb-like architecture), wet keratin (ghost-like nests of
keratinocytes)

Germ Cell Tumors


o Clinical:
Children/young adults
Pineal gland (boys), suprasellar (both sexes)
Results in: Parinauds syndrome (upward gaze paralysis) and
precocious puberty
Germinoma = radiosensitive, teratoma = resectable; otherwise worse
prognosis
o Radiograph: homogenously enhancing mass

Return to Table of Contents

82


o Pathology:
Dual cell population mitotically active clear tumor cells (primordial
germ cells) + small reactive lymphocytes

CNS Lymphoma
o Clinical
Elderly (non-EBV) or immunosuppressed (EBV)
Deep/periventricular, single mass or multifocal
Initially steroid responsive; poor prognosis; treat w/ methotrexate
o Pathology
Angiocentricity (invasion of vessel walls by large atypical cells
prominent nucleoli and high N:C ratio)
Primarily B-Cells (CD-20 stain)

Schwannoma (Grade I)
o Clinical
Return to Table of Contents

83

At cerebellopontine angle vestibular schwannoma


Spinal cases intradural extramedullary or dumbbell-shaped
extending through foramina
Eccentric growth preserves parent nerve function
NF2 implicated (sporadic and familial)
If non-vestibular consider schwannomatosis
o Pathology
Biphasic tumor dense Antoni A and loose Antoni B
Dense nuclear palisading Verocay bodies

Describe the WHO grading scheme for CNS neoplasms and recognize the features of
low vs. high-grade neoplasms
Benign = grade 1; malignant = grade 2, 3, 4
Low-grade = grades 1, 2; high-grade = grades 3, 4 (differences in treatment)
Recognize the key histologic features that distinguish one tumor from another
Look for underlined key words in first objective
Identify the typical location of the various CNS tumors
Adult CNS Tumors: supratentorial (2/3)
o Most common tumors in adults = metastasis
o Most common primary tumor in adults = meningioma
o Most common malignant primary tumor in adults = GBM
o Ependymoma spinal cord
Pediatric CNS Tumors: infratentorial (2/3)
o Most common benign pediatric = pilocytic astrocytoma
Cerebellum, hypothalamus/3rd ventricle, spinal cord
Optic pathway strongly consider neurofibromatosis 1
o Most common malignant pediatric = medulloblastoma
o Also ependymoma
More commonly in 4th ventricle, rarely supratentorial
o Ganglioglioma temporal lobe

Return to Table of Contents

84

Headache
Distinguish between primary and secondary headaches
Primary not caused by an underlying disease
o Migraines (w/ or w/o aura)
o Cluster headache
o Indomethacin-responsive headaches
o Tension-type headache
Secondary resulting from an underlying disorder
o Thunderclap headache
o Headache associated w/ vascular disorders
o Headache associated w/ brain tumor, trauma
o Headache associated w/ infection
o Headache associated w/ trigeminal neuralgia
o Headache associated w/ substance use or withdrawal
Define the clinical picture of migraine, cluster headache, indomethacin-responsive
headaches, tension-type headache, and headache associated with subarachnoid
hemorrhage, giant cell arteritis, trigeminal neuralgia, and meningitis
Migraine
o Criteria for Diagnosis
5+ attacks lasting 4-72 hrs
2+ of the following: unilateral, pulsating, moderate-severe intensity,
aggravated by routine physical activity
1+ of the following: nausea and/or vomiting, photophobia and
phonophobia
No evidence of disease that might cause headaches
o Four phases: prodrome, aura, headache, postdrome

Prodrome: can be excitatory or inhibitory in origin; stereotyped


pattern that occurs before every migraine; ideal to start treatment at
this phase
Aura: complex of focal neurologic symptoms made up of positive or
negative phenomena
Visual (most common) scotoma (blind spot), fortification
spectra (zig-zag line pattern), scintillations
Return to Table of Contents

85

Motor hemiparesis
Sensory numbness, dysesthesias
Speech
Headache (described above) in children, may resolve w/ sleep
Resolution/Postdrome
Mood changes (most common)
Muscular weakness
Physical tiredness
Reduced appetite
Cluster Headache (6 males: 1 female)
o Intense, unilateral pain (orbital, supraorbital, temporal)
o 15-180 minutes; occur in clusters once, twice, several per day for period of
one or months before going into remission
o Autonomic abnormality ipsilateral to headache conjunctival injection,
lacrimation, ptosis, miosis, eyelid edema, nasal congestion, rhinorrhea, facial
sweating
o May become chronic 10%; later onset = higher likelihood
Indomethacin-Responsive Headaches
o Chronic Paroxysmal Hemicrania (5 women: 1 man, onset 30 y/o)
Severe unilateral orbital, supraorbital, temple pain
Up to 45 minutes, may have 4-40 attacks/day
Autonomic symptoms present; precipitated by head flexion/rotation
o Benign Cough Headache (middle aged men)
Transient, severe, explosive head pain upon coughing, sneezing,
weight-lifting, bending, stooping
Evaluate with neuroimaging (look for posterior fossa abnormality)
o Coital Headaches (4 men: 1 woman, onset 20-60 y/o)
Sudden, severe, throbbing, occipital headaches just before or during
orgasm (after orgasm = migraine)
Infrequent, intermittent; not psychogenic
o Sharp Stabbing Headache
Ice pick pains; brief (< 3 seconds) in V1 ditribution
Can move from one side to another; common w/ migraine
Tension-Type Headache
o Lasts 30 minutes-days; not pulsating; bilateral
o Not worse with physical activity; mild-moderate but not disabling
o No associated symptoms or possible photophobia/phonophobia, but no N/V
Thunderclap Headache
o Sudden onset of severe head pain a/w neck stiffness
o Subarachnoid hemorrhage ruptured aneurysm or vascular malformation
Giant Cell Arteritis
o Vasculitis affecting older patients (>50) mostly women (3:1)
o Continuous throbbing pain (temples/occiput) + temporal artery tenderness
with diminished or absent pulse
o Polymyalgia rheumatic fever weight loss, jaw claudication, aching and
stiffness in the neck, shoulder girdle, and pelvic girdle
Return to Table of Contents

86

o Risk of blindness
Trigeminal Neuralgia
o Brief, electric shock-like/lancinating facial pain
o Affects women more than men (3:2)
o V2 and V3 > V1 divisions affected
o Triggered by trivial stimuli
o Usually unilateral (consider demyelination like MS if bilateral)
Meningitis
o Severe, generalized (not focal) headache + neck stiffness
o Vomiting

Define current hypotheses for the pathogenesis of migraine


Vasoconstriction/Vasodilation Hypothesis not correct
Spreading Cortical Depression (spreads posterior to anterior)
o Diffuse neural dysfunction vasodilation of cerebral blood vessels
o Activation of trigminovascular system from irritation to trigeminal nerve
endings by vasodilation
o Release of vasoactive neuropeptides (CGRP, substance P) exacerbate
vasodilation
o Pain impulses from dilation and inflammation are transmitted to trigeminal
nucleus caudalis in brain stem

o
Outline strategies for abortive and prophylactic therapy of migraine and cluster
headache
Abortive Therapy
o 5-HT1B/1D receptor agonists Triptan
Appropriate 1st line therapy for moderate severe migraines
Highly specific no affinity for other serotonergic, adrenergic, or
dopaminergic receptors
o DHE 45 IV or NS preparations available
o Valproate sodium IV
Return to Table of Contents

87

o NSAIDs no more than 10-15 days/month (avoid MOH)


o Short taper w/ prednisone, dexamethasone or longer-acting triptan
o Specific to Cluster Headache
Oxygen inhalation, corticosteroids
Prophylactic Therapy
o Eliminate triggers (sleep, food, stress, menses, medications)
o Medication indicated with:
Recurring disabling migraine refractory to acute treatment
Frequent attacks (>2/week) risk of medication overuse
Problems with acute medications
Patient preference physicians, airline pilots
Presence of uncommon migraine conditions
Hemiplegic migraine
Migraine w/ prolonged aura
Basilar migraine
Migrainous infarction
o Anticonvulsants (valproate, gabapentin, topiramate, zonisamide,
levetiracetam)
o Antidepressants (TCAs, SSRIs, SSRI/SNRI, MAOIs)
o Beta-blockers (propranolol)
o Calcium channel agonists (verapamil) particularly for migraine w/ aura
o Others: NSAIDs, riboflavin, magnesium, butterbur, Botox injections

Return to Table of Contents

88

Neurobiology of Rehabilitation
Describe the main impairments in a patient with a focal cerebral lesion
Direct destruction of groups of neurons or regions of the brain specialized for
different functions
Lesion-Symptom Mapping
Region of the Brain
Deficit
Motor Cortex
Hemiparesis
Visual Cortex
Hemianopia
Sensory Cortex
Hemianesthesia
Language cortex frontal
Production aphasia
Language cortex temporal
Receptive aphasia
Prefrontal cortex
Executive function, apraxia
Parietal cortex
Attention, apraxia
Hippocampus
Memory: long-term
Fronto-parietal
Memory: short-term
Disconnection of fibers that connect a region of the brain with another region of the
brain or the spinal cord
o Diaschisis remote metabolic effects measure with SPECT/PET
o Functional connectivity abnormalities in dynamic interactions
o Idea of network-level damage
Describe the time course of recovery after brain injury
Recovery of Motor Function
o Varies with severity of initial deficit
o Plateau in function is reached by 3 months
Recovery of Aphasia
o Takes longer than motor function evidence of recovery at 1 year
o The degree of initial deficit is highly predictive of outcome
Define mechanisms through which lesions cause behavioral deficits and recovery of
function
Diaschisis
o Decrement of rCBF or glucose metabolism in structurally normal regions
connected with site of damage
Abnormal recruitment of additional regions of activation
o Normally task may only drive motor cortex and SMA contralaterally
o Signal change in one bran region inversely correlates with motor function
lesion can cause multiple regions of abnormal activation

Return to Table of Contents

89

o
Network-wide dysfunction of inter-regional interactions
o Abnormal activity and synchronicity of regions functionally connected to the
damaged area
o Cross-inhibition is important for synchronization of areas
Local Recovery Mechanisms
o Synaptic sprouting
o Axonal regrowth
o Functional remapping
Long-Range Mechanisms
o Resolution of remote metabolic dysfunction
o Growth of new pathways
o Rebalancing of activity at the network level

Distinguish between primary effects of brain lesions and complications


Most common complications
o Dysphagia
o Bladder abnormalities (incontinence or retention) and infections
o Deep venous thrombosis
o Spasticity
o Seizures
o Arrhythmias
o Autonomic instability
Relate how mechanisms of recovery may be related to rehabilitation interventions
Stroke/Brain Injury Unit Admission
o Decreases mortality and disability
o Fluid status, temperature, blood pressure control, amount of mobilization,
team approach
o Enrichment, reward, measurement-based approach
Task-specific Training
o Amount of improvement increases with more time
o Training is better given in short amount of time than over longer periods of
time
o Task-specific are more effective than non-specific exercises
Return to Table of Contents

90

o 20 hrs of therapy should be given to improve any behavior (more training for
legs than for upper extremities is necessary)
Drugs fluoxetine, amantadine, methylphenidate
Constraint Induced Therapy (CIMT)
o Improves motor function in chronic stroke patients extensive task-specific
training and shaping while wearing a mitten or glove to prevent movements
of normal arm
o Train on repetitive goal-oriented movements of upper extremity for 4-6
hrs/day for 2-week period
Cognitive rehabilitation speech and language therapy in stroke

Return to Table of Contents

91

Aphasia
Differentiate the primary language disorders (aphasias) and distinguish them from
disturbances of speech production (dysarthria)
Dysarthria inability or difficulty to pronounce or articulate words
Oral apraxia problem in planning movements of the tongue, lips, & pharynx during
speech
Aphasia syndromes of disordered language caused by brain damage
o Problems in speaking or writing words in sentences
o Problem in understanding spoken or written words
To differentiate: ask patient to write down words or sentences they cannot
pronounce or speak aphasic patients make same errors regardless of modality
Dysphonia voice disorder; weakness of laryngeal muscles
Mutism inability to produce any verbal utterance caused by disordered language,
speech, or voice
Given the clinical scenario of a patient with aphasia, predict the anatomical
localization of the responsible lesion

Brocas Aphasia
o Left inferior frontal lobe
o Nonfluent slow, effortful
o Agrammatical
Difficulty understanding complex grammatical structure
o Pronunciation errors
Phonological paraphasia word sounds alike
Semantic paraphasia word is related by meaning
o Preserved language comprehension
Simple conversation
Carry out simple commands
o Writing is impaired in similar way to speech
o Weakness of contralateral face/arm may accompany
Wernickes Aphasia
Return to Table of Contents

92

o Posterior left superior temporal gyrus


o Impaired language comprehension inability to transform incoming verbal
auditory stimuli into symbolic information
o Fluent speech production is normal
o Many speech errors
Substitutions of letters or words paraphasias
Meaningless words neologisms
Jargon aphasia
o Impaired repetition comprehension deficits
o Paresis/sensory loss typically does not occur
Conduction Aphasia
o Arcuate fasciculus connects Wernickes and Brocas
o Deficit in repetition
Global Aphasia
o Entire perisylvian region is damaged
o Input, output, repetition are impaired
o Right hemiparesis usually present
Transcortical Motor Aphasia
o Anterior extrasylvian region prefrontal & medial frontal cortex
o Difficulty in initiating speech
o Repetition is spared
Transcortical Sensory Aphasia
o Posterior extrasylvian inferior temporo-occipital cortex
o Impaired language comprehension
o Repetition is spared
Mixed Transcortical Aphasia
o Entire extrasylvian region is involved
o Only remaining language skill is repetition

Discuss prognostic factors and mechanisms of aphasia recovery


No effect in adults gender, age, handedness
Children (<12 y/o) remarkable recovery of language function
Thrombotic/embolic strokes tend to recover less (permanent loss of cortical
neurons in ischemic injury)
Larger lesions less likely to recover
Speech therapy (intensity and duration) are critical to recovery
Mechanisms Responsible
o Restoration of normal resting blood flow & metabolism in distant regions
o Reactivation of regions near the lesion
o Recruitment of homologous regions in the right hemisphere

Return to Table of Contents

93

Neglect
Describe the functional deficits associated with spatial neglect
Spatial Bias
o Automatic orienting of eyes to the right w/ bilateral stimuli
o Extinction to double simultaneous visual/tactile stimulation
Lack of awareness for stimuli presented in the left visual field or left
side of body
o Spontaneous rotation of eyes and body to right at rest
o Rightward exploratory manual bias, even blindfolded
o Motor neglect of left hand, though strength is relatively spared
Impaired Vigilance, Arousal, Sustained Attention
o Meditated by a different neural mechanism
o But interacts with the spatial attention function transient increases in
vigilance can improve spatial bias and vice versa
Insight and Body Awareness Deficits
o Anosognosia: lack of concern about acute change in functional status
o Hemisomatognosia: lack of recognition of weakness
o Somatomotor Paraphrenia: incorrect attribution of stimuli
o Represents disconnection between parts of brain that report subjective
experience and those pars that code for the body or motor actions
Discuss the pathophysiologic principles of spatial neglect
Bias is multimodal underlying dysfunction involves widely distributed sensory and
motor representations across the brain independent of vascular distribution
o Reflects not just structural damage, but metabolic/neural disruptions on
distributed cortical/subcortical systems
Spatial bias includes negative and positive deficits disruption of normal
interhemispheric balance of activity
o Reciprocal interaction between L & R sides of CNS
o Balanced activity is necessary for gaze, head, and body positions that are on
average centrally aligned
o Bias is independent of sensory information or actions
o Present at multiple levels of CNS
Rightward spatial bias can be modified at any moment by the same internal or
external signals that modulate healthy brain balance
o Severity of hemispatial neglect is not fixed; can be dynamically modified
o Voluntary direction of attention, decreasing saliency of stimuli on the right,
increasing activity in the damaged brain, limiting movement of normal hand
Describe the principles of rehabilitation for spatial neglect
Increase awareness & use of left body
o Task-specific exercises performed with left hand
o Right hand placed in mitten during therapy and meals
Visual Scanning Therapy
o Retrains search impairment in neglect
Return to Table of Contents

94

o Does not generalize well to other tasks


Prismatic Adaptation
o Prisms induce pointing error in direction of light displacement
o Adaptation occurs within 10-15 minutes
o Effect is extinguished in 10 minutes in healthy individuals
o In neglect patients, 1 hour of improvement for multiple deficits results
o Repeated exposures have longer lasting effects
Optokinetic Stimulation
o Large fields of stripes/dots moving right to left to induce smooth pursuit eye
movements
o Transiently improves neglect - active tracking induces rebalancing of
saliency signals
TMS and tDCS Stimulation
o Enhance activity of the right hemisphere
o Suppress activity of the left hemisphere
Computer Game Stimulation
o Training sustained attention/inhibitory control through computer games

Return to Table of Contents

95

Multiple Sclerosis
Describe the most common clinical presentations of multiple sclerosis
Initial Symptoms (keep in mind that these are initial symptoms when someone
presents for medical attention, not necessarily the very first symptom)
o Limb weakness 40%
o Decreased vision/optic neuritis 22%
o Tingling/paresthesia 21%
o Double vision/diplopia 12%
o Dizziness/vertigo 5%
o Urinary bladder urgency, frequency, hesitancy 5%
White Matter Signs
o Bilateral INO
o Hyper-reflexia, clonus
o Spasticity
o Babinski response
o Posterior column sensory deficits (vibration > proprioception)
o Cerebellar intention tremor, dysmetria
o Optic Neuritis
Optic disc pallor, Marcus-Gunn pupil
Red desaturation/decreased light brightness
Central scotoma
Uthoffs phenomenon worse vision with heat
List the four clinical subtypes of MS
Relapsing-Remitting MS (RRMS) most common clinical subtype
Primary-Progressive MS (PPMS)
Secondary-Progressive MS (SPMS)
Progressive-Relapsing MS (PRMS)

Return to Table of Contents

96

Discuss the epidemiology and genetics of MS


Epidemiology
o Age of onset: 15-45 y/o (peak around 30)
o Gender: 70% women
o Geography: incidence increases with distance from equator
o Incidence: 12,000 new cases per year
o Prevalence: 0.1% of US population
o Race: Caucasians > other ethnic groups
Genetics
o Likely polygenic inheritance
o Having a family member (or multiple) increases risk
o MHC association: HLA-DR2 (chr 6p21.3), explains 20-50% of genetic
susceptibility
o Relative risk of 2 3 for DRB1*1501 (if homozygous, more severe course
with earlier onset) and DRB1*1503
o Other genes: IL-2RA, IL-7RA, HLA-DRA
Immune Involvement
o Evidence female predominance, autoimmune animal model
o Molecular Mimicry viral peptide with resemblance to myelin basic protein
peptide
Environment indicated by epidemiologic evidence, geographic clustering,
migration data
o EBV infection confers higher risk

o Vitamin D risk of MS decreases with higher baseline levels of vit D


Prognostic Indicators (on a population level)
o Good: optic neuritis at onset, sensory onset, little disability at 5 years,
relapsing/remitting, full recovery from attacks
o Poor: cerebellar dysfunction, high attack rate, motor symptoms at onset,
progressive course, male gender

Discuss the diagnosis criteria for MS, including clinical and laboratory findings
typical of MS
Multiple discrete lesions in the CNS separated by time and space, without an
alternative explanation
Return to Table of Contents

97

McDonald Criteria (for clinical trials)


o Dissemination in Time
No interval requirement for T2 lesion (previously needed at least 2
MRIs for diagnosis)
MRI: simultaneous presence of asymptomatic Gd+ lesion and nonenhancing lesions at any time
o Dissemination in Space
Swanton MRI Criteria (2006) simpler and more practical
o Dissemination in Time: new T2 lesion on follow up MRI irrespective of timing
relative to baseline MRI scan
o Dissemination in Space one or more T2 lesions in at least two of four
Barkhof locations
Juxtacortical, periventricular, infratentorial, spinal cord
MRI
o Multiple white matter lesions (>3 mm)
o Corpus callosum involvement may lead to atrophy
o Dawsons fingers: perpendicular orientation to ventricles (same pattern of
distribution as venules that drain the brain)
o Ovoid shape
o Gadolinium enhancement
o Short (less than 3 segment) spinal cord lesions not complete transverse
o Atypical MRI Findings consider alternative diagnosis
Tumor-like mass lesions or exclusively punctate lesions
Anterior temporal lobe & occipital lobe involvement
Gray matter lesions (w/o white matter) cortex, thalamus
Longitudinally extensive lesions in spinal cord (> 3 segments)
Sparing of corpus callosum
Diffusion restriction
o Remember extent of MRI lesions does not correlate well with symptoms
o Things that DO correlate with disability
Black hole burden reflect severe axonal injury

Brain atrophy
Magnetic resonance spectroscopy (decreased N-acetylaspartate)
Spinal Fluid Analysis
Return to Table of Contents

98

Nucleated cell count: <5/mm 3 in 75%; <40/mm3 in 90%


Glucose: normal
Protein: normal in 60%; >100mg/dl is very rare
Increased myelin basic protein not specific; does not correlate w/
prognosis or severity
o Oligoclonal bands (IgM and IgG) - > 90%
Indicate that IgG was synthesized in the CSF
B cells localize to brain to generate antibodies

o
o
o
o

Not entirely specific consider many infectious diseases


o Increased CSF index (85-90%)
o Increased intrathecal IgG, IgM, IgD, IgA
Evoked Potentials
Biopsy (occasionally needed)
o Mononuclear inflammatory cels
o Demyelination
o Relative axonal sparing
o Astrocyte hypertrophy
o Spinal cord involvement, with cervical spinal cord disproportionately
involved
o Periventricular lesions
o Gray matter lesions cortical lesions

Discuss therapeutic options available now, and their effects on disease


Beta-interferons
o Reduce exacerbations
o Reduce disability
o Slows time from 1st visit to development of SPMS
Natalizumab anti-VLA-4/alpha4 integrin
o Decreases the risk of relapse
o Reduces the number of new lesions on MRI
o Higher risk of progressive multifocal leukoencephalopathy (PML)
Anti-JC virus antibody status (+ = greater risk)
Prior immunosuppression (+ = greater risk)
Length of treatment w/ Natalizumab (longer = greater risk)
Fingolimod first oral therapy
o Reduces relapse rate significantly
o Reduces disease progression
o Safety greater efficacy with greater risks
Return to Table of Contents

99

Transient reduction in heart rate on treatment initiation


Elevated blood pressure
Elevated liver enzymes
Macular edema
Skin cancers
Herpes infections
Two fatal infections when treated with high dose
Hemorrhagic encephalitis with high dose
Posterior reversible encephalopathy syndrome with high dose
Teriflunomide first line agent
o Anti-proliferative and anti-inflammatory
o Inhibits pyrimidine synthesis enzyme dihydroorotate dehydrogenase
(DHODH) that is required for T and B lymphocyte proliferation
o Reduces the number of active MRI lesions
o Reduces annualized relapse rate
o Reduces disease progression
o Safety/Contraindications
Hepatotoxicity (contraindicated in hepatic impairment or in patients
taking leflunomide)
Teratogenicity (contraindicated in pregnancy)
Can be eliminated using cholestyramine or activated charcoal
Notable side effects: GI discomfort, hair thinning, liver blood test
abnormalities, weight loss
Dimethyl Fumarate/BG-12 oral capsule taken BID
o Enhances nuclear factor erythroid 2 related factor 2 (Nrf2) pathway
o Anti-inflammatory
o Reduced relapse rate, reduced disability progression, reduced enlarged T2
lesions
o Adverse Events
Flushing (hot flashes), GI (diarrhea, nausea, upper abdominal pain)
No increase in serious infections or malignancies
One PML incident
Mean lymphocyte count reduced
Increase liver enzymes
Treatment of Relapses
o Corticosteroid treatment shortens attack, does not alter severity of
impairment
o IV methylprednisolone may transiently delay further attacks in patients with
optic neuritis
Symptomatic Treatment
o Fatigue, depression, cognitive dysfunction
o Neurogenic pain, nystagmus, tremor
o Bladder, bowel, sexual dysfunction
o Spasticity, spasms, cramps, tonic spasms
o K+ channel blockers (has seizure risk) improved walking

Return to Table of Contents

100

Pathology of Diseases of Myelin


Describe the production/maintenance of CNS and PNS myelin and identify the
microscopic appearance of the oligodendroglial cell
CNS myelin
o Maintained by oligodendrocytes
o Oligodendrocytes are responsible for multiple internodes of myelin (30-50)
on multiple axons
o These cells are fragile; if lost, results in more diffuse injury; do not produce a
basal lamina (CNS nodes of Ranvier are naked)
o Oligodendrocytes proliferate in response to injury, but does not normalized
structure/function
PNS myelin
o Maintained by Schwann cells (1 Schwann cell:1 axon)
CNS vs. PNS Myelin different myelin associated proteins & diseases

o
Oligodendroglia fried egg artifact; located in white matter

o
Describe the gross and microscopic pathology of multiple sclerosis and how these
correlate with clinical symptomatology and can be used to understand disease
pathogenesis, create and interpret animal models, and develop rational forms of
therapy
Gross Pathology:
o Inactive plaque: sharp borders, gray color; tend to be periventricular

Return to Table of Contents

101


o Dawsons fingers projections of demyelination along blood vessels

o Involvement of the optic nerves/chiasm optic neuritis

o Plaques may be present on the brainstem or spinal cord (cross boundaries)

o Less commonly intracortical MS plaques (target gray matter)

Return to Table of Contents

102

Microsopic Pathology
o Prominent cellularity with a border of normal white matter

o Macrophages contain undigested fragments of myelin in active plaques

o Prominent mononuclear cell infiltrate in parenchyma & perivascular region


Parenchyma: CD8, CD4 T cell
Return to Table of Contents

103

Perivascular: T-cells and some B-cells


Recall: Th1 cells important for cell-mediated immunity role in
inflammatory demyelination

o Ultrastructural: direct stripping of normal myelin naked axons with


preserved oligodendrocytes

o Reactive astrocytes in active MS plaques

o Shadow plaques areas of remyelination thin myelin sheaths

Return to Table of Contents

104


o Chronic/Inactive plaques sharp-edged myelin loss with minimal
inflammation & macrophages; relatively little cellularity

Acute (Marburg type) Multiple Sclerosis


o Large, ill-defined, diffuse patterns of myelin loss

o
o Perivascular lymphocytes, parenchymal macrophages, reactive astrocytes
o Marked cellularity with axonal swellings from parent axons (tadpoles)

o
Return to Table of Contents

105

Disease Pathogenesis
o Most common pattern is type II antibody-mediated demyelination
o T-cells and complement perivascular plaques
o Reduction in MBP, MOG, MAG, PLP myelin proteins
o Relative sparing of oligodendroglia (early on)
o Extensive remyelination
o Resembles autoimmune encephalitis (EAE sensitized with MOG model)
Experimental animals immunized with whole myelin or selected
constituents inflammatory demyelinating disease of CNS
o Response to plasma exchange
o MRI ring enhancement with gadolinium breach of BBB
o Multiple Factors: autoimmune, genetics, environment (virus)
Vitamin D deficiency
History of EBV infection
Cigarette smoking

Describe the clinical presentation, pathologic manifestations, and likely


pathophysiologic mechanism of acute disseminated encephalomyelitis
Clinical Presentation:
o Acute, monophasic demyelinating disease in children or adults
o Typically follows inciting event
Viral infection (measles, mumps, varicella, EBV)
Bacterial infection (mycoplasma, C. jejuni)
Immunizations (smallpox, rabies)
Pathologic Manifestations
o Little gross pathology; mild discoloration & edema of subcortical white
matter

o
o Multiple perivascular foci of demyelination (spinal cord)

o
o Inflammation results in little axon loss
Return to Table of Contents

106

Pathophysiologic no identified predisposing cause, likely immune-mediated

Describe the histopathology of progressive multifocal leukoencephalopathy (PML),


immune reconstitution inflammatory syndrome (IRIS), HIV leukoencephalopathy,
and HIV-associated vacuolar myelopathy
Progressive Multifocal Leukoencephalopathy (PML)
o Clinical:
Complication of underlying immunosuppression
If untreated, 3-12 month course; improved if immunosuppression is
reversed
Motor, sensory, personality changes dementia & death (w/o
treatment)
o Pathology:
Often symmetric, may produce asymmetric pattern

Patches of myelin loss extend into cerebral gray


No sparing of subcortical U-fibers
Very little perivascular inflammation

Enlarged plum-colored oligodendrocyte nucleus


Stick-and-ball papova virus particles in oligodendrocyte nucleus

JC virus may transform astrocytes


Immune Reconstitution Inflammatory Syndrome (IRIS)
Return to Table of Contents

107

o Clinical
Imbalance of CD4+/CD8+ T lymphocytes
White matter attack by JC-virus specific CD8+ cytotoxic T cells
May have continued or absent JC virus infection
o Pathology
Massive infiltration of parenchyma by CD8+ T cells
Myelin destruction w/ T lymphocytes
Macrophages & microglial cells diffusely distributed/forming cuffs
HIV Leukoencephalopathy
o Pathology
Gross discoloration and atrophy; cortical atrophy

Myelin pallor with relative U-fiber sparing


Pallor, astrocytosis, multinucleated giant cells in white matter

Vacuolar Myelopathy
o Pathology
Peculiar myelin vacuolation + substantial axon loss

Return to Table of Contents

108

Describe the general neuropathologic features of the leukoencephalopathy and


contrast the histologic appearance of metachromatic leukodystrophy, globoid cell
leukodystrophy, adrenoleukodystrophy, and Alexander disease
General Pathologic Features
o Confluent areas of myelin and oligodendroglial loss
o Relative axonal sparing
Metachromatic Leukodystrophy (autosomal recessive)
o Loss of arylsulfatase A accumulation of sulfatide (myelin component)
o Confluent gelatinous areas of white matter damage; spares gray matter

o
o Loss of myelin with relative sparing of U fibers; numerous macrophages

o
o Brown deposits stored material is metachromatic
o Prismatic inclusions in macrophages, oligodendroglia, Schwann cells

o
o Demyelinated axons; Schwann cells have stored sulfatide; macrophages
accumulate sulfatide; may result in onion-bulb appearance
Krabbe Disease (autosomal recessive)
o Loss of galactocerebrosidase abnormal degradation of myelin
o Global and confluent myelin loss with U-fiber sparing
Return to Table of Contents

109

o
o Multinucleated globoid cells in perivascular distribution
Adrenoleukodystrophy (X-linked recessive)
o Accumulation of very long chain fatty acids deficiency of peroxisomes
o Myelin loss preferentially affects occipital lobes

o
o Perivascular lymphocytic infiltration
o Gene therapy using lentiviral vector with wild-type ABCD1
Alexander Disease
o Gene defect in GFAP, but not hereditary
o Discolored, sunken-in white matter with exaggeration of Rosenthal fibers

Return to Table of Contents

110

Movement Disorders and Therapy


Describe and recognize the clinical manifestations of Parkinson disease; recognize
features suggestive of other disorders associated with Parkinsonism
Resting Tremor
o Relatively slow (3-5 Hz) tremor, commonly asymmetric
o Involves hands, lips, head, leg
o Worse with stress, improves with sleep
o Abates with action, can be seen when arm is at side while walking
o Sometimes tremor re-emerges with action
Bradykinesia
o Slowness of amplitude and rate of movement with repetitive movement
o Micrographia handwriting starts large but gets progressively smaller
o Usually asymmetric at first; worse with stress
Akinesia
o Paucity of spontaneous movement
o Mask-like expression, statue-like
o Severity does not correlate with that of bradykinesia
o May give appearance of depression
Rigidity
o Uniformly increased resistance to a passive range of motion
o Patient may complain of stiffness and slowness
o Usually symmetric
o Rigidity increases with volitional activity (opening/closing the other hand)
o Cog-wheeling: ratcheting with rigidity; not specific to PD
Postural Instability
o Difficulty maintaining balance with perturbation
o More pronounced forward/backward; side-to-side is intact
o Functionally important and impairing
Gait/Posture
o Increased flexion worse on affected side (may present w/ shoulder
impingement)
o Small, shuffling gait; affected side may drag
o Turn en bloc shoulders and hips move together
o Freezing difficulty initiating walking (lack of internal cues)
Oropharyngeal Problems
o Speech Abnormalities hypophonic, dysarthric speech (soft, monotone)
o Swallowing Problems eat slowly, difficulty moving food from front to back
of throat; at risk for silent aspiration
Autonomic Problems
o Drooling
o Bowel/bladder problems
o Orthostasis
o Sexual dysfunction
o Heat intolerance/sweating
o Convergence problems
Return to Table of Contents

111

Psychiatric Problems
o Mood disorders
o Anxiety disorders
o Psychosis hallucinations
Associated Conditions
o REM behavior disorder (may antedate PD) acting out dreams
o Restless legs
o Reduced olfaction (and thereby taste)

Discuss the pathophysiology of Parkinson disease and, from this information, the
differential diagnosis of Parkinsonism
Alpha-synuclein deposition
o Begins in caudal brainstem and ascends
o May contribute to development of dementia
o A-beta protein may or may not be present (if present faster progression)

o
o Autosomal dominant mutation on chromosome 4 identified protein
o Most common gene defect is LRRK2 (autosomal dominant)
0.5% of PD have this defect
Pockets: North African Arabs (30-40% risk), Ashkenazi Jews (19%)
DA neuron loss
Striatal DA loss
o Loss of 25-30% of nigrostriatal reserve results in parkinsonism
DDx Parkinsonism
o Multisystems Atrophy
Nigrostriatal degeneration symmetric Parkinsonism
Cerebellar problems
Autonomic nervous system involvement can be severe
Synucleinopathy, glial cytoplasmic inclusions
Doesnt respond to medications
Differentiate from PD with pathology
o Progressive Supranuclear Palsy
Symmetric Parkinsonism
Supranuclear gaze palsies (difficulty moving eyes down)
Speech/swallowing problems are severe
Rigidity: axial (neck) > appendicular
Falling occurs early in the course
Dementia
o Corticobasal Degeneration
Asymmetric Parkinsonism
Limb movement problems: apraxia, dystonia, myoclonus, alien limb
syndrome
Return to Table of Contents

112

Supranuclear gaze palsies


Speech/language problems: early and severe
Dementia
Pathology: tauopathy in glia and neurons

Explain the rationale for pharmacotherapy of Parkinson disease including treatment


with levodopa, dopamine agonists, and surgical approaches; discuss the common
side effects of anti-parkinsonian medications
Levodopa + Carbidopa
o Levodopa crosses BBB to exert CNS effects
o Carbidopa inhibits peripheral AADC to prevent L-DOPA conversion to
dopamine in the periphery (side effects nausea/vomiting + orthostasis)
o Early/Mild PD: small amount of L-DOPA has big effect
o Duration of benefit decreases as disease progresses
MAOB Inhibition: Selegiline, Rasagiline

o
COMT Inhibition: entacapone, tolcapone
o Blocks metabolism of dopamine in brain and periphery
o Tolcapone is very potent but can cause fatal liver damage

o
Dopamine Agonists
o Bromocriptine, pergolide
o Pramipexole daytime somnolence/insomnia
Return to Table of Contents

113

Ropinirole GI upset
Rotigitine transcutaneous administration
Apomorphine rapid action; injection; nausea & vomiting
Common Side Effects
Orthostasis
Psychosis
Impulse control disorders
Daytime sleepiness
Dystonia & Dyskinesia can occur as drug responses on period, peak period, off
period (on and off = diphasic)
o Amantadine 60% reduction for dyskinesia; modest benefit in Parkinsonism
Levodopa Gel via PEG continuous infusion gives smoother effect profile
Deep Brain Stimulation
o Target: subthalamic nucleus
o Use microelectrodes and electrical activity to identify STN
o STN: motor, cognitive, mood regions
o
o
o
o

Distinguish essential tremor from Parkinsonism and discuss the basic approach to
treatment of essential tremor
Characteristics of Essential Tremor
o Usually symmetric; commonly affects hands (also head/neck, voice)
o Head no-no movement
o Exacerbated with sustained posture causes functional disability
o Faster frequency (3-8 Hz) though this is dependent on age
o Frequently relieved by alcohol
Epidemiology
o Age of Onset: 45 y/o, peak in 2nd and 6th decades
o Equal incidence in men and women
o Associated syndromes: dystonia, Parkinsonism, myoclonus
o Familial (60-70%): autosomal dominant with variable penetrance
Treatment
o Primidone
o Propranolol (peripheral beta blockade)
o Topiramate
o Alcohol
o Thalamic stimulation
Recognize the basic clinical manifestations of dystonia and how different parts of the
body may be affected
Dystonia: syndrome of sustained muscle contractions, frequently causing twisting
and repetitive movements or abnormal postures
Axis 1 Features tend to go together
o Age of onset infancy, childhood, adult onset
o Body part affected
o Temporal pattern rapid or slow onset
o Associated features isolated or combined
Return to Table of Contents

114

Axis 2 Features
o CNS pathology neurodegeneration, structural lesion, none apparent
o Inheritance inherited, acquired, idiopathic
Cranial Dystonia
o Mean age onset: 52
o 72% female
o Can affect eyes excessive blinking, squeezing of eyes
Oromandibular Dystonia
o Opening (or closing of the jaw)
o Excessive movement of the tongue
Lingual Dystonia
o No control of the tongue
o Impairs ability to speak
o Common sensory trick: straw in the mouth
Cervical Dystonia
o Mean age onset: 45
o 75% female
o Twisting of the neck
Retrocollis backwards movement
o Common sensory trick: touching the face
Hand Dystonia
o Mean age onset: 40
o 55% female
o Tightness of grip difficult to write
o Sensory trick: touching the wrist
Task specific Dystonia
o Age of onset: 28, 56 (examples given in class)
o For highly skilled, repetitive motions
o Occurs in 1-2% of professional musicians
Laryngeal Dystonia
o Mean age onset: 50
o 80% female
o Adductor strangled speech; normal voice with whispering
o Abductor very breathy; sudden, explosive breaths when speaking
Dystonia gait
Generalized Dystonia
o More common in children (adult onset tends to be focal)
o All parts of body are involved (proximal and distal)
Primary Torsion Dystonia
o Multiple genetic mutations associated
o TOR1A, THAP1, CIZ1, ANO3, GNAL
Dopa Responsive Dystonia
o GTP6 cyclohydrolase defect (autosomal dominant) more common
o Tyrosine hydroxylase defect (autosomal recessive)
o Administration of dopa has dramatic improvement

Return to Table of Contents

115

o
Explain the mechanism of action of botulinum toxin and how it helps dystonia
Botulinum A: cleaves SNAP 25
Botulinum B: cleaves synaptobrevin
Work at NMJ cleave docking proteins chemical denervation

Side Effects:
o Local excessive weakness
o Distant effects: autonomic, exacerbation of underlying condition (MG)
o Resistance

Recognize the clinical manifestations of adult-onset and juvenile-onset Huntingtons


disease and understand the genetic basis of this disease
Genetic Basis
o Autosomal dominant
o CAG expansion in IT15 gene, chromosome 4
o Polyglutamine incorporated in huntingtin
o Normal: <26
o Normal, but mutable child may be at risk: 27-35
o Intermediate repeat length senile chorea: 36-39
o HD CAG repeat length (very long = younger onset): >40
o Genetic testing is diagnostic and can be done pre-symptomatically
Tends to have good psychiatric impact, even when test shows person
is at increased risk of HD

Return to Table of Contents

116

o
Clinical Manifestations
o Motor Abnormalities
Chorea involuntary jerky movements frequently incorporated into
other movements
Dystonia
Parkinsonism bradykinesia, rigidity, postural instability
o Dysarthria/Dysphagia
o Oculomotor abnormalities
Decreased volitional saccades
Saccadic intrusions
Blink to break fixation
o Cognitive Problems
Bradyphrenia slowness of thinking
Dementia
Executive functioning abnormalities
Planning
Sequencing problems
o Behavioral Problems
Personality changes irritability, apathy, etc
Depression: 30-50%; suicide risk
Psychosis: 10% lifetime risk
Juvenile Onset
o Occurs before 20 y/o
o 80-90% paternal inheritance
o Clinical manifestations dystonia, bradykinesia (doesnt start as chorea)
Pathophysiology
o Atrophy of caudate and putamen
o Cortical atrophy
o Loss of certain cell populations:
Medium-sized spiny neurons
GABA projection neurons
Return to Table of Contents

117

Encephalin neurons (early)


Substance P neurons (later)
o Large & medium aspiny interneurons are spared
Treatment
o Mostly supportive education, counseling, etc
o GABA agonists: valproate, clonazepam
o Presynaptic dopamine depleters
Tetrabenazine (only FDA approved Rx, $40,000/yr)
Alpha-methylparatyrosine
Reserpine
o Dopamine receptor blockers
o Symptom Specific Treatment
Parkinsonism levodopa, dopamine agonists
Dystonia levodopa, dopamine agonists, botulinum toxin, baclofen
Depression SSRIs, etc
Irritability mood stabilizers, neuroleptics
Psychosis - neuroleptics

Identify the major dopamine receptor antagonist-induced movement disorders


including acute dystonic reactions, tardive dyskinesia, and tardive dystonia
Acute/Subacute Reactions
o Dystonia
Oculogyric crisis twisting of chin with eyes moving up
May also involve the tongue
If severe posturing of whole trunk
o Akathisia
Motor restlessness accompanied by inner sense of restlessness
o Parkinsonism
o Neuroleptic Malignant Syndrome
Clinical diagnosis: autonomic dysfunction & fever; Parkinsonism with
severe rigidity; mental status changes
Pathophysiology: dysfunction of DA pathways DA receptor
blockade; presynaptic depletion; dopa withdrawal
Treatment; supportive care, DA agonists, paralysis
Tardive Reactions
o Dyskinesia
Age of onset: older
Permanent: likely
Anticholinergics: worsen
Stereotypies tongue protrusion and lip smacking; vacuous chewing
movements; belly dancer dyskinesia
o Dystonia
Age of onset: younger
Permanent: less likely than dyskinesia
Anticholinergics: may improve
Involuntary movements grimacing of face with hand movements
Return to Table of Contents

118

Involvement of shoulders, intermittent involvement of diaphragm


Internal rotation of upper extremities
o Akathisia
o Tics
o Treatment of Tardive Reactions
Reduce exposure
Presynaptic dopamine depletion
Benzodiazepines
Describe the proper treatment for acute dystonic reactions
Administer anticholinergic
o Benztropine
o Diphenhydramine
Do NOT give benzodiazepines

Return to Table of Contents

119

Dementia
Discuss the impact of Alzheimers disease (AD) form an epidemiologic perspective
Prevalence increases with age (dramatic increase after age 65)
Projected number of patients is estimated to be 14.3 million in 2050 (aging
population)
Currently 5th leading cause of death in people > 65 y/o
Unlike other major diseases, AD deaths rose by 66% in 2000
Define dementia of the Alzheimer type and characterize its clinical presentation,
course, and underlying neuropathology
Clinical Presentation
o Neurodegenerative brain disorder, regardless of clinical status, representing
a continuous process of synaptic and neuronal deterioration
o Defined by intraindividual cognitive decline, from subtle to severe, that
interferes with daily function, and can be subclassified on symptom severity
Incipient (prodromal; mild cognitive impairment)
Dementia
o Interference with everyday function
o Memory deficits usually prominent
o Other cognitive and non-cognitive impairment
Disorientation; impaired judgment and problem solving; inattention
Personality change (withdrawal, delusions)
Course (~7-10 years)
o Two major stages:
Preclinical (presymptomatic; asymptomatic)
Symptomatic/clinical

o Gradual onset and progression


o CDR: clinical dementia rating
Incipient (<0.5): insidious onset
Mild (CDR 1): forgetful, repetitious, impaired decisional abilities;
independent in self-care, looks and acts normal, can perform some
IADLs, but often impaired to some degree

Return to Table of Contents

120

Moderate (CDR 2): only highly learned material recalled; little or no


pretense of IADLs; disruptive behaviors; supervised BADLs
Severe (CDR 3): oriented only to self; requires full care for BADLs;
akinetic mutism
Neuropathology
o Plaques (diffuse, neuritic, CAA) composed of amyloid-beta protein; located
in neuropil; disrupts neuronal activity
o Neurofibrillary tangles (NFT) composed of tau protein; located in neurons
o Neuronal and synaptic dysfunction and loss atrophy of brain
o Inflammation

Describe diagnostic guidelines for dementia due to AD and diagnostic tools including
the Ascertain Dementia 8 (AD8)
Definition: an acquired syndrome of decline in memory and other cognitive domains
sufficient to affect daily function
Detection:
o Intra-individual change: informant observations about decline in previously
established cognitive and functional abilities
This is more indicative of cognitive decline dementia
o Inter-individual differences: cognitive test performance compared with ageand education-matched norms
This can be problematic because there is a great deal of variation in
baseline performance across individuals
AD8 questionnaire reports only changes caused by memory & thinking difficulties:
o Is there repetition of questions, stories, or statements?
o Are appointments forgotten?
o Is there poor judgment (e.g., buys inappropriate items, poor driving
decisions)?
o Is there difficulty with financial affairs (e.g., paying bills, balancing
checkbook)?
o Is there difficulty in learning or operating appliances (e.g., television remote
control, microwave oven)?
o Is the correct month or year forgotten?
o Is there decreased interest in hobbies and usual activities?
o Is there overall a problem with thinking and/or memory?
Describe the differential diagnosis of the demented patient, including the relevance
of specific clinical features and the utility of laboratory and neuroimaging
procedures
Rapidly Evolving Dementias
o Temporal profile
o Laboratory results
Vascular Dementia
o Result of stroke
o Present with focal signs
Lewy Body Dementia
Return to Table of Contents

121

o Extrapyramidal symptoms
o Visual hallucinations
Frontotemporal Dementia
o Behavior and language changes

Describe the efficacy of currently available drugs for the symptomatic treatment of
AD and discuss potential disease-modifying strategies
Cholinesterase Inhibitors
o Donepezil
Adverse Effects: nausea, vomiting, diarrhea, dizziness, muscle cramps,
vivid dreams
Useful at all stages
o Galantamine
Adverse Effects: nausea, vomiting, diarrhea, dizziness, headache
Used for mild to moderate AD
o Rivastigmine
Adverse Effects: nausea, vomiting, diarrhea, dizziness, drowsiness,
headache
Used for mild to moderate AD
Memantine NMDA receptor antagonist
o Adverse Effects: constipation, dizziness, headache, nonspecific pain
o Used for moderate to severe AD
Important to initiate treatment early!
Recognize the major protective and risk factors for AD, including the role of
causative mutations and genetic susceptibility factors
Older Age
Family History/Genetics
o Chromosomal disorder: all Down Syndrome individuals develop AD
o Rare single gene mutation: APP, PSEN1, PSEN2 w/ autosomal dominant
o Complex: most AD cases result from mixture of genetic susceptibility (APOE
apolipoprotein E) and environmental risk factors
No current evidence to support association of any modifiable factor
o Associations do not imply causality
o Large studies are needed to establish these associations
Discuss the psychosocial impact of dementia on patients and their families
Impact on Patients
o Neuropsychiatric symptoms increase morbidity and caregiver distress
o Agitation, anxiety, irritability, psychosis (delusions and hallucinations),
depression
o Risk of wandering
o Behavior modification techniques are equally effective as antipsychotics and
antidepressants
Impact on Families
o Caregiver profile 75% are female relatives
Return to Table of Contents

122

o Alternatives paid caregivers, long term care


o Chronic stress, poor health, overwhelmed, frustration, depression, futility,
bereavement are common
Describe biological markers for AD, relevance in clinical setting, and caveats for
practical use
Enhance diagnostic confidence for AD
o Tests must be standardized and validated
o Understanding the tests sensitivity/specificity is necessary before informing
medical decision-making
o Potential uses: diagnosis in symptomatic persons; screening in asymptomatic
persons; prognosis
Molecular Biomarkers of AD
o Low levels of CSF A-beta 42
o Elevated levels of CSF tau and phosphor-tau
o Amyloid imaging
PET + C PIB

PT + F Florbetapir
Clinical amyloid imaging
Recently approved in April 2012
Other tracers: flutemetamol (2013), florbetaben, tau
Downstream Indicators of Neurodegeneration
o Reduced temporoparietal metabolism with FDG-PET
o Whole brain and/or regional atrophy on MRI
o Disruptive connectivity on fMRI
Causative gene mutation
o PSEN1, PSEN2, APP
o APOE4 insufficiently specific to be a diagnostic biomarker

Return to Table of Contents

123

Pathology of Neurodegenerative Disease


Describe the common features of neurodegenerative diseases
Neuronal loss and gliosis (disease of glial cells in some)
Selective vulnerability of neurons
Signature inclusions
o Abnormal protein conformation, folding, aggregation, propagation
o Abnormal protein degradation
o Axonal transport defects
o Oxidative stress & free radicals (metabolically active)
o Apoptosis/programmed cell death
Describe the neuroanatomic distribution of neuronal loss in the most common
neurodegenerative diseases and the resulting neurological symptoms
Symptom
Typical Disease
Anatomy
Dementia
Alzheimers Disease
Cerebral cortex
Chorea
Huntingtons Disease
Basal ganglia (striatum)
Parkinsonism
Parkinsons Disease
Nigrostriatal pathway
Ataxia
Friedreichs Ataxia
Cerebellum/spinal cord
Motor abnormalities Amyotrophic lateral sclerosis Upper/lower motor neurons
Discuss the neuropathological features of:
Alzheimers Disease
o Risk Factors: age, family history, genetics
APP (21q21) amyloid precursor protein
PSEN1 (14q24.3) enzyme involved in AB42 production
PSEN2 (1q31-q42) enzyme involved in AB42 production
Trisomy 21 overproduction of APP
APOE-e4 allele: late onset (>65), sporadic slower clearance of AB42
o Pathogenesis

Formation of AB42 monomers


Synaptic activity
Trisomy 21, PS1, PS2, APP mutation
AB42 monomers form AB oligomers (and then plaques) if not cleared
APOE genotype
o Gross Features
Thinning of gyri + widening of sulci
Temporal > parietal > frontal > occipital (mostly spared)
Atrophy of hippocampus as well (learning and memory)
Cortical atrophy ventricular dilatation hydrocephalus ex vacuo
Return to Table of Contents

124


o Histopathological Features
Marked neuronal loss & gliosis; numerous reactive astrocytes

Silver stain: neuritic plaques & neurofibrillary tangles (flame-shaped)


Plaques = extracellular, beta-pleated sheets
Tangles = intracellular tau protein resistant to degradation
(can serve as tombstones ghost tangles)

Evolution of Plaques

A-beta peptide deposition occurs in vessels amyloid angiopathy


Return to Table of Contents

125

Impairs physiological responses (weak, prone to hemorrhage)

Can demonstrate with Thioflavin S fluorescence microscopy


Immunohistochemistry
PHF1 antibody labels hyperphosphorylated tau protein

Granulovacuolar degeneration in pyramidal neurons of hippocampus

Hirano bodies not specific for AD


o New Imaging Techniques
PET-PIB PIB preferentially adheres to plaques
Frontotemporal Dementia
o Clinical: 5-15% of dementia (3rd leading cause); younger presentation
Personality change, social disinhibition, mutism
Episodic memory & visuospatial function relatively preserved
Prototype = Picks disease
o Gross Features
Frontotemporal atrophy ends abruptly at precentral gyrus

o Histopathological Features

Return to Table of Contents

126

Pick bodies: round, tau-immunopositive cytoplasmic inclusions


(dentate gyrus, amygdala, cortex)

Neuropil threads, thorn-shaped astrocytes


Coiled bodies oligo inclusions; ballooned neurons in cortex
Can be tagged with ubiquitin

TAR DNA-binding protein 43 (TDP-43) major component of inclusions


Superficial neuron loss, gliosis, vacuolation

Huntingtons Disease
o CAG triplet expansion polyQ (glutamine) in Huntingtin, chr 4p16.3
o Gross Findings: marked atrophy of caudate nuclei

o Histopathological Findings
Marked neuronal loss & gliosis of caudate + reactive astrocytes
Return to Table of Contents

127

Intranuclear striatal neuronal inclusions stain brown


Parkinsons Disease
o Gross Findings: pallor of substantia nigra

o Histopathological Findings
Lewy bodiesalpha-synuclein; halo in locus ceruleus/brainstem nuclei

Cortical Lewy bodies may not have a halo


Amyotrophic Lateral Sclerosis
o Combined UMN/LMN disease; 1-3/100,000 per year
o 5-10% familial genes linked to ALS-FTD (including hexanucleotide repeat)
o Gross Findings:
Relative atrophy of ventral roots compared to dorsal roots
Atrophy of precentral gyrus (uncommon finding)

o Histopathology:
Loss of motor neurons in spinal cord, brain stem, cortex
Degeneration of corticospinal tracts
Return to Table of Contents

128

Atrophy of anterior spinal roots


Neurogenic atrophy (grouped atrophy) of muscles

Neuronal inclusions (TDP-43, ubiquitin positive)


Round hyaline inclusions (Lewy body-like)

Bunina bodies (irregular, eosinophilic), Skein-like inclusions


Friedreichs Ataxia
o Degeneration of sensory nerves (DRG), posterior columns, Clarkes column,
spinocerebellar & pyramidal tracts, Dentate nucleus, superior vermis
o Ataxia + absent LE reflexes, hypertrophic cardiomyopathy (death)
o Autosomal recessive; onset 10 yrs
o GAA repeat disorder of FRDA1 gene on 9q13 (NL = 7-55 copies)
Aberrant heterochromatin formation less frataxin protein
Iron accumulates in mitochondria (frataxin binds iron, delivers to
iron-sulfur cluster machinery cells/MT import more iron)
o Histopathological Findings:
Marked pallor of degenerated posterior columns (gracile>cuneate)

Return to Table of Contents

129

Creutzfeldt-Jakob Disease
o Prototypic prion disease
Rapid dementia (<1 year duration)
Myoclonus
Periodic spikes on EEG
MRI: basal ganglia lesions
LP: positive CSF protein 14-3-3 + CSF tau protein
o Sporadic (90%, peak 65 y/o), familial (5-10%, PRNP mutation), iatrogenic,
new variant (ingestion of prions)
PrP = prion protein; PrPc = normal (alpha-helical)
PrPSc/CJD/res = pathogenic, protease resistant, transmissible
Risk factor for sporadic: codon 129 genotype (methionine vs. valine)
MM = 37% of population, 74% of cases
MV = 51% of population, 15% of cases impeded propagation
of protein misfolding
VV = 12% of population, 11% of cases
New Variant linked to mad cow disease
Younger age of onset (28 y/o)
Sensory/psychiatric symptoms, ataxia
MRI: T2/proton-weighted signal in posterior thalamus
Extensive PrP amyloid plaques (w/ MM genotype only)
Transmission is likely via lymphoid tissues (little in skeletal
muscle)
o Gross Pathology: normal to variable cerebral atrophy
o Histopathological Findings
Spongiform change, neuronal loss, gliosis (cortex, basal ganglia,
thalamus are most common locations)

IHC important for diagnosis; can also do Western blot

Distinguish tauopathies from alpha synucleinopathies


Return to Table of Contents

130

Tauopathy: inclusions of tau protein


o Picks disease
Synucleinopathy: inclusions of alpha-synuclein protein
o Idiopathic Parkinsons disease (IPD)
o Diffuse Lewy body disease (DLBD)
o Multiple systems atrophy

Return to Table of Contents

131

CNS Infections
List the predominant etiologic organisms in bacterial meningitis of the neonate,
child, and adult
Adult (>5 years): Streptococcus pneumoniae, Neisseria meningitides, Listeria
monocytogenes
Children (1 month 5 years): H. flu type B (reduced with vaccination),
streptococcus pneumoniae, Neisseria, GBS
Neonates (< 1 month): Group B streptococcal meningitis, E. coli, Listeria

Describe the symptoms and signs, CSF profile, and management of bacterial, fungal,
and viral meningitis; encephalitis; brain abscess; and subdural or epidural abscess
Signs of Meningitis
o Headache
o Fever
o Stiff neck Brudzinski sign (flex neck), Kernigs sign (flex hip)
o Confusion
o Stupor/coma
o Seizures
Bacterial Meningitis
o Hints:
Meningococcal: adolescents, crowded living conditions
Pneumococcal: young, old adults, splenectomized, sickle cell
HIb: non-vaccinated, day care, family members
Trauma/instrumentation: staph, nosocomial pathogens
Recurrent: importance of CSF rhinorrhea as clue
o CSF Profile
Pressure: elevated (> 180mm CSF)
Cell count: markedly elevated
Normal is < 5 nucleated cells
Often 250 or more w/ PMN predominance
Glucose: low (Normal >2/3 serum), often < 40 mg/dL
Return to Table of Contents

132

Protein: high (Normal <45 mg/dl), often >100mg/dL


Gram stain: may show causative organisms
Cultures are important for deciding treatment
o Empiric Therapy (adults/children): 3rd-gen cephalosporin (ceftriaxone,
cefotaxime) + vancomycin (antibiotic-resistance)
Ampicillin (+/- gentamicin) in critically ill or if Listeria is suspected
o Initial therapy (neonates): ampicillin + ceftriaxone/cefotaxine (+/gentamicin)
o Nosocomial: vancomycin + cephalosporin for pseudomonas (ceftazidime)
o Focus when the organism is identified
o Fluid restriction for SIADH (expected)
o Corticosteroids (controversial use)
Start before or with first dose of antibiotics
Parenteral dexamethasone 0.15/mg/kg q6 h x 4 days
Reduces hearing loss in children
Reduces mortality, especially in adults w/ streptococcus pneumoniae
Fungal Meningitis
o Features
Insidious, subacute course
Variable intensity of symptoms
Tend to localize around brainstem basilar meningitis and
predominance of brainstem signs
o CSF Findings
Pressure: variably elevated
Protein: elevated
Glucose: suppressed
Cells: <1000/mm3, organism on smear and in culture
o Cryptococcal antigen is reliable, rapid test
o Possible organisms
Cryptococcus neoformans immunosuppressed/HIV
Candida opportunistic, burns, TPN
Aspergillus opportunistic, presents as vasculitis
Coccidiomycosis recent travel to desert SW
Histoplasmosis Ohio & Mississippi River basin
Mucormycosis diabetic, steroid, immunosuppression
o Treatment
IV amphotericin B (significant side effects)
Flucytosine with ampB
Fluconazole
Itraconazole alternatives
Repeat LP, monitor pressure
Role of corticosteroids is uncertain
Should consider TB meningitis
Viral Meningitis
o Clinical Picture localized meningitis; may be called aseptic
o CSF Findings
Return to Table of Contents

133

Pressure: normal
Cell count: mildly elevated (10-200, lymphocytes predominate)
Glucose: normal
Protein: normal, slightly high but generally <100 mg/dl
No organisms on gram stain or culture
o Organisms
Coxsackie, ECHO, mumps, polio, HSV-2, lymphochoriomengitis
Abscess
o Clinical Picture
Focal brain signs (motor/sensory/speech deficit)
Headache, confusion/drowsiness, seizures
Fever is unreliable
May also consider brain tumor
CT/MRI shows contrast rim enhancing pattern & surrounding edema
o Organisms:
Associated with infections elsewhere in body
Spread via bone or veins from sinus/ear
Hematogenous spread: acute bacterial endocarditis, sepsis
Streptococci, anaerobes
Mixed flora, fungi, actinomyces, nocardia, toxoplasma (AIDS)
o Treatment
Surgical sampling + mass reduction + prolonged antibiotics
Encephalitis
o Clinical Picture
Focal brain signs + aggressive course and fever
More widespread/multifocal brain manifestations
May cause mass effect w/ lateralized brain swelling
Confusion/mental status change; seizures
o CSF Findings
Pressure: increased
Cell count: increased WBC (lymphocytic predominance) + RBC
(hemorrhagic encephalitis)
Protein: elevated
Glucose: normal or low
PCR can identify HSV-1
IgM antibodies diagnose West Nile encephalitis
o Organisms:
HSV acute and aggressive course (prototypic)
Arboviruses West Nile, St. Louis, Eastern equine, California
Prions
o Treatment:
HSV acyclovir (genetics TLR3 allele interferon response)
West Nile CCR5-delta-32 protect from HIV, but more virulent WNV;
supportive care

Discuss the causative agent and clinical picture of Creutzfeldt-Jakob Disease


Return to Table of Contents

134

Causative agent: protein folding abnormality resulting in deposit of neurotoxic


proteinaceous material
o Transmission of protein (no nuclei)
o Can be sporadic, hereditary, infectious
Clinical Picture
o Personality change, psychiatric change
o Motor problems, myoclonus, progressive cognitive disease
o Death in months years

Discuss the clinical picture, treatment, and complications of herpes zoster infection
of peripheral nerve
Latent infection with reactivation later in life
Discuss the neurological conditions associated with primary HIV infection and the
secondary neurological complications of HIV infection
HIV enters brain almost immediately controlled by immune system
Primary HIV conditions
o HIV-associated neurocognitive impairment (HAND)
Concentration, memory, speech problems
Motor slowing
Behavioral change
o Myelopathy
o Peripheral neuropathy
Complications
o CMV reactivation causes aggressive disease (encephalitis, radiculomyelitis,
neuritis); PCR testing; ganciclovir, foscarnet, cidofovir
o PML (JC virus) acquired demyelinating CNS disease (DNA virus);
progressive focal deficits
o Toxoplasma encephalitis ingested from poorly cooked meat that is latent in
45% of adults; reactivation causes multifocal encephalitis; treat w/ sulfa
drugs
o Neurosyphilis diagnose with CSF evaluation
o Tuberculosis abscess or meningitis; difficult to treat
o Primary CNS lymphoma (associated with EBV) aggressive mass lesion;
treat with XRT, HAART
o Cryptococcal meningitis

Return to Table of Contents

135

Pathology of CNS Infections


Discuss the unique aspects and problems associated with CNS infections

Recognize the morphologic patterns of CNS infection (e.g., meningitis, encephalitis,


abscess)
Intraparenchymal
o Abscess, mass, encephalitis, myelitis
Meningitis (more commonly leptomeningeal > pachymeningeal)
Ventriculitis or choroid plexus
Vascular involvement vasculitis/vasculopathy infarction; aneurysm
Discuss the clinicopathologic manifestations and potential complications of specific
pathogens
Acute Bacterial Meningitis
o Gross Appearance
Opacification of leptomeninges from pus
Prominent venous congestion in regions of inflammation

o Histological Appearance
Expansion of subarachnoid space by neutrophils
Cortex is not involved pia mater as protective barrier

Secondary vasculitis (with thrombus)


o Complications
Arterial/venous infarcts
Seizure disorder
Cognitive deficits
Return to Table of Contents

136

Developmental delays
Deafness
Fibrin and collagen deposition in subarachnoid space

Can lead to blockage of outflow tracts hydrocephalus

Cerebral Abscess
o Hematogenous (septic emboli) often in MCA territory
o Mass effect (may be mistaken for tumor)
o Underlying pathology: endocarditis, CHD, IV drugs, periodontal disease,
dental work, sinus infection
o Gross Appearance
Necrotic center hyperemic edge of granulation tissue reactive
astrocytosis surrounding entire area

o Histological Appearance
Center: necrotic debris, neutrophils, bacteria (gram-stain)

Return to Table of Contents

137

CNS Tuberculosis
o Hematogenous source, most commonly at base of brain
o Secondary vasculitis/cranial nerve palsies
o Tuberculoma = mass lesion in parenchyma
o Potts disease = spinal involvement with vertebral body collapse
o Gross Appearance
Necrotizing granulomas w/ rare bacilli

o Histological Appearance
Multinucleated giant cells; rare acid-fast bacilli (thin, red rods)

Cryptococcus Meningitis (fungal CNS)


o Gross Appearance
Infection tracks along Vrichow-Robin spaces (perivascular extensions
of subarachnoid space)
Dilated spaces soap-bubble appearance

Return to Table of Contents

138

o Histological Appearance
Mucin stain mucopolysaccharide capsule
PAS organism with clear halo that does not stain
Lack of inflammation

Candida Microabscess
o Gross Appearance
Tendency to lodge in cortex & gray-white matter junctions

o Histological Appearance
Yeast & pseudohyphae (spaghetti & meatballs)

Return to Table of Contents

139

Fungal Granuloma
o Coccidioides, Histoplasma

o
o Coccidioides spherules in cytoplasm of multinucleated cells

CNS Aspergillus
o Fungal hyphae invade thrombosed vessels
o Septated hyphae with acute angle branching (>90* = mucor)

o
Viral Encephalitis
o Perivascular lymphocytic inflammation
o Microglial nodules
o Microglial activation (CD68 IHC)

Return to Table of Contents

140

o
Herpes Encephalitis
o Gross Appearance
Hemorrhagic/necrotizing
Bilateral, but usually asymmetric
Prefers temporal lobes (can see atrophy as consequence)
o Histological Appearance
Cowdry type A inclusions intranuclear inclusions; eosinophilic core
with clear halo
Cowdry type B inclusions ground-glass appearance

Cysticercosis
o Most common cause of epilepsy in Mexico
o Due to larval form of Taenia solium (when organism dies immune response
symptoms)
o Gross Appearance
Cyst with intracystic larva (can be seen on imaging)

Return to Table of Contents

141

May be seen at the gray-white matter junction


o Histological Appearance
Larva: highly corrugated integument
Scolex head of parasite

Describe the clinicopathologic manifestations of HIV disease of the CNS, including


opportunistic infections, HIV encephalitis, and vacuolar myelopathy
HIV encephalitis/ADC
o Clinical: cognitive, behavioral, motor deficits; mild cerebral atrophy
o Histological Appearance
Infected macrophages + multinucleated giant cells

Vacuolar Myelopathy
o Myelin pallor/vacuolation of posterior & lateral columns
o Mimics B12 deficiency
o Numerous foamy macrophages + tissue vacuolation + myelin loss

o
CMV Infection (opportunistic)
o AIDS patient & neonates (TORCH organism)
o Gross Appearance
Ventriculitis, choroid plexitis, encephalitis
o Histological Appearance
Return to Table of Contents

142

Nuclear (Owls eye, common) & cytoplasmic (rare) inclusions


Microcephaly, chorioretinis, CNS malformations

PML (reactivation of JC Virus)


o Gross Appearance
Acute demyelination; multiple foci
Loss of white color & granular friable lesions

o Histological Appearance
Foamy CD68+ macrophages, especially at margins
Oligodendrocytes (1st to be infected) plum-colored inclusions
Reactive astrocytes w/ eosinophilic cytoplasm (may have
incorporated viral genome atypical appearance)

CNS Toxoplasmosis (opportunistic infection)


o AIDS/neonates (TORCH infection)
o Gross Appearance
Necrotizing encephalitis
Often presents as intraparenchymal abscess
o Histological Appearance
Return to Table of Contents

143

Free and encysted organisms in tissue


Encysted = bradyzoite
Free = tachyzoite actively replicating form

Return to Table of Contents

144

Вам также может понравиться